teksty zadań - Olimpiada Matematyczna

advertisement
TEKSTY ZADAŃ
Zawody stopnia pierwszego
1. Rozwiązać w liczbach rzeczywistych równanie
|x| − |x + 2| + |x + 4| − |x + 6| + . . . − |x + 998| =
= |x + 1| − |x + 3| + |x + 5| − |x + 7| + . . . − |x + 999| .
2. Na bokach AB i AC trójkąta ABC zbudowano, po jego zewnętrznej
stronie, kwadraty ABDE i ACF G. Punkty M i N są odpowiednio środkami
odcinków DG i EF . Wyznaczyć możliwe wartości wyrażenia M N : BC.
3. Wykazać, że liczba
10
10
X
n=0
!
2 · 1010 n
10
5 jest podzielna przez 22·10 −1 .
2n
4. Dowieść, że wykres wielomianu W (x) stopnia większego od 1 ma oś
symetrii wtedy i tylko wtedy, gdy istnieją takie wielomiany F (x) i G(x), że
W (x) = F (G(x)), przy czym G(x) jest stopnia 2.
5. Wykazać, że dla każdej liczby naturalnej k istnieje taka liczba naturalna m, że każda z liczb m, 2m, 3m, . . . , m2 w rozwinięciu dwójkowym ma
dokładnie k jedynek.
6. Okrąg dzieli każdy bok rombu na 3 odcinki. Malujemy otrzymane
odcinki kolejno na czerwono, zielono i biało, zaczynając od wierzchołka rombu
i poruszając się po jego obwodzie w ustalonym kierunku. Wykazać, że suma
długości odcinków czerwonych jest równa sumie długości odcinków białych.
7. W grupie n ­ 3 osób każda ma parzystą (być może zerową) liczbę
znajomych. Dowieść, że istnieją trzy osoby mające tę samą liczbę znajomych.
Uwaga: Zakładamy, że nikt nie zalicza siebie samego do grona swoich
znajomych, oraz że osoba A zna osobę B wtedy i tylko wtedy, gdy B zna A.
8. Niech S(n) oznacza sumę cyfr liczby n. Dowieść, że dla dowolnej liczby
naturalnej n liczba S(2n2 + 3) nie jest kwadratem liczby całkowitej.
9. Płaszczyzna przecina krawędzie boczne graniastosłupa prawidłowego
sześciokątnego tworząc w przekroju sześciokąt wypukły D1 D2 . . . D6 . Niech di
(1 ¬ i ¬ 6) będzie odległością punktu Di od płaszczyzny ustalonej podstawy
graniastosłupa. Dowieść, że
d21 + d23 + d25 = d22 + d24 + d26 .
25
10. Dana jest szachownica 2000 × 2000. Na każdym polu leży kamień.
Wykonujemy następujące ruchy: jeśli na pierwszym i trzecim z trzech kolejnych pól leżących w jednym wierszu lub kolumnie leży kamień, to możemy
oba te kamienie przełożyć na drugie z tych pól (niezależnie od tego, czy jakiś
kamień leży na środkowym polu, i czy ruch opróżni jakiekolwiek pole).
Rozstrzygnąć, czy można wykonując takie ruchy przełożyć wszystkie kamienie na jedno pole.
)B > <
) C. Punkt D leży na od11. Dany jest trójkąt ABC, w którym <
) DAC = 21 (<
)B −<
) C) . Okrąg styczny w punkcinku BC i spełnia równość <
cie A do prostej AC i przechodzący przez punkt D przecina prostą AB
w punkcie P 6= A. Dowieść, że
BP BD
=
.
AC DC
12. W niemalejącym ciągu a1 , a2 , a3 , . . . wszystkie wyrazy są liczbami
całkowitymi dodatnimi, a ponadto dla każdego k dokładnie k wyrazów jest
równych k. Wyznaczyć wszystkie liczby pierwsze postaci
a1 + a2 + . . . + an .
Zawody stopnia drugiego
1. Dana jest taka funkcja f : R → R, że dla każdej liczby rzeczywistej x
zachodzą równości
f (x) = f (2x) = f (1 − x) .
Dowieść, że funkcja f jest okresowa.
2. W czworokącie wypukłym ABCD zachodzą równości
<
) ADB = 2<
) ACB oraz <
) BDC = 2<
) BAC .
Udowodnić, że AD = CD.
3. W n-osobowym stowarzyszeniu działa sześć komisji. W skład każdej
z nich wchodzi nie mniej niż n/4 osób. Dowieść, że istnieją dwie komisje oraz
grupa licząca nie mniej niż n/30 osób, należących do obu tych komisji.
4. Wyznaczyć wszystkie takie trójki liczb pierwszych p ¬ q ¬ r, że liczby
pq + r,
pq + r2 ,
qr + p,
qr + p2 ,
rp + q,
rp + q 2
są pierwsze.
) BAC = 90◦ , jest podstawą ostrosłupa
5. Trójkąt ABC, w którym <
ABCD. Ponadto zachodzą równości
AD = BD
◦
) ACD ­ 30 .
Udowodnić, że <
26
oraz
AB = CD .
6. Wyznaczyć wszystkie takie liczby naturalne n, że dla dowolnych liczb
rzeczywistych x1 , x2 , . . . , xn , y1 , y2 , . . . , yn zachodzi nierówność
x1 x2 . . . xn + y1 y2 . . . yn ¬
q
x21 + y12 ·
q
x22 + y22 · . . . ·
q
x2n + yn2 .
Zawody stopnia trzeciego
1. Wyznaczyć wszystkie takie trójki liczb naturalnych a, b, c, że liczby
a2 + 1 i b2 + 1 są pierwsze oraz
(a2 + 1)(b2 + 1) = c2 + 1 .
2. Na bokach AC i BC trójkąta ostrokątnego ABC zbudowano, po jego
zewnętrznej stronie, prostokąty ACP Q i BKLC o równych polach. Udowodnić, że środek odcinka P L, punkt C oraz środek okręgu opisanego na trójkącie
ABC leżą na jednej prostej.
3. Na tablicy są napisane trzy nieujemne liczby całkowite. Wybieramy
z tej trójki dwie liczby k, m i zastępujemy je liczbami k +m i |k −m|, a trzecia liczba pozostaje bez zmiany. Z otrzymaną trójką postępujemy tak samo.
Rozstrzygnąć, czy z każdej początkowej trójki liczb całkowitych nieujemnych,
kontynuując to postępowanie, można otrzymać trójkę, w której co najmniej
dwie liczby są zerami.
4. Dowieść, że dla każdej liczby naturalnej n ­3 i dla każdego ciągu liczb
dodatnich x1 , x2 , . . . , xn zachodzi co najmniej jedna z nierówności
n
X
xi
n
­ ,
x + xi+2 2
i=1 i+1
n
X
xi
n
­
x + xi−2 2
i=1 i−1
(przyjmujemy xn+1 = x1 , xn+2 = x2 oraz x0 = xn , x−1 = xn−1 ).
5. W przestrzeni dany jest trójkąt ABC oraz sfera s rozłączna z płaszczyzną ABC. Przez każdy z punktów A, B, C poprowadzono prostą styczną
do tej sfery. Punkty styczności oznaczono odpowiednio K, L, M . Punkt P
leży na sferze s i spełnia warunki
AK BL CM
=
=
.
AP
BP
CP
Udowodnić, że sfera opisana na czworościanie ABCP jest styczna do sfery s.
6. Dana jest liczba naturalna k. Określamy ciąg (an ) wzorami
a1 = k + 1,
an+1 = a2n − kan + k dla n ­ 1 .
Wykazać, że jeżeli m 6= n, to liczby am i an są względnie pierwsze.
27
XLII Międzynarodowa Olimpiada Matematyczna
1. W trójkącie ostrokątnym ABC punkt O jest środkiem okręgu opisanego. Punkt P leżący na boku BC jest spodkiem wysokości z wierzchołka A.
) BCA ­ <
) ABC + 30◦ . Wykazać, że
Zakładamy, że <
<
) CAB + <
) COP < 90◦ .
2. Udowodnić, że
√
a
a2 + 8bc
+√
b
b2 + 8ca
+√
c
c2 + 8ab
­1
dla dowolnych liczb rzeczywistych dodatnich a, b, c.
3. W zawodach matematycznych uczestniczyło 21 dziewcząt i 21 chłopców. Okazało się, że:
• Każdy zawodnik i każda zawodniczka rozwiązał(-a) co najwyżej sześć zadań.
• Dla każdej dziewczyny i każdego chłopaka istnieje co najmniej jedno zadanie, które zostało rozwiązane przez oboje z nich.
Udowodnić, że istnieje zadanie, które zostało rozwiązane przez co najmniej
trzy dziewczyny i co najmniej trzech chłopców.
4. Niech n będzie liczbą całkowitą nieparzystą większą od 1 i niech
k1 , k2 , . . . , kn będą danymi liczbami całkowitymi. Dla każdej spośród n! permutacji a = (a1 , a2 , . . . , an ) zbioru {1, 2, . . . , n} przyjmijmy
S(a) =
n
X
ki ai .
i=1
Dowieść, że istnieją dwie różne permutacje b oraz c takie, że liczba S(b)−S(c)
jest podzielna przez n!.
5. W trójkącie ABC dwusieczna kąta BAC przecina bok BC w punkcie P , a dwusieczna kąta ABC przecina bok CA w punkcie Q. Wiadomo, że
<
) BAC = 60◦ oraz AB + BP = AQ + QB. Wyznaczyć możliwe miary kątów
trójkąta ABC.
6. Niech a, b, c, d będą liczbami całkowitymi spełniającymi warunki
a > b > c > d > 0. Zakładamy, że
ac + bd = (b + d + a − c)(b + d − a + c).
Dowieść, że liczba ab + cd nie jest liczbą pierwszą.
28
XXIV Austriacko-Polskie Zawody Matematyczne
1. Wyznaczyć liczbę całkowitych dodatnich wartości a, dla których istnieją takie nieujemne liczby całkowite x0 , x1 , x2 , . . . , x2001 , że
ax0 =
2001
X
axk .
k=1
2. Niech n będzie liczbą całkowitą większą niż 2. Rozwiązać w liczbach
rzeczywistych nieujemnych x0 , x1 , x2 , . . . , xn następujący układ równań:
xk + xk+1 = x2k+2 ,
k = 1, 2, . . . , n,
gdzie xn+1 = x1 i xn+2 = x2 .
3. Niech a, b, c będą długościami boków trójkąta. Udowodnić, że
a + b b + c c + a a3 + b3 + c3
+
+
−
¬ 3.
2<
c
a
b
abc
4. Udowodnić, że jeśli a, b, c, d są długościami kolejnych boków czworokąta (niekoniecznie wypukłego) o polu S, to spełniona jest nierówność
1
S ¬ (ac + bd).
2
Rozstrzygnąć, dla jakich czworokątów ta nierówność staje się równością.
5. Odbywamy zamkniętą podróż skoczkiem po szachownicy, odwiedzając każde z 64 pól dokładnie jeden raz i numerując kolejno odwiedzane
pola liczbami od 1 do 64. Następnie wybieramy dodatnie liczby rzeczywiste x1 , x2 , . . . , x64 . Dla każdego białego pola z numerem i określamy
yi = 1 + x2i −
q
3
x2i−1 xi+1 ,
a dla każdego czarnego pola z numerem j określamy
yj = 1 + x2j −
q
3
xj−1 x2j+1 ,
gdzie x0 = x64 i x65 = x1 . Udowodnić, że
64
X
yi ­ 48.
i=1
6. Dla ustalonej dodatniej liczby całkowitej k rozważamy ciąg (an )n­0
√
zdefiniowany następująco: a0 = 1, an+1 = an +d k an e dla każdego n ­ 0, gdzie
dxe oznacza najmniejszą liczbę całkowitą nie mniejszą niż x. Dla każdego
k ­ 1 wyznaczyć zbiór Ak składający się ze wszystkich całkowitych wartości
√
ciągu ( k an )n­0 .
7. Rozważmy dodatnie liczby całkowite N , w których rozwinięciu dziesiętnym nie występuje cyfra 0 i których suma cyfr S(N ) jest dzielnikiem N .
(a) Udowodnić, że wśród rozważanych liczb N istnieje nieskończenie wiele
takich, w których rozwinięciu dziesiętnym każda z cyfr od 1 do 9 występuje tyle samo razy.
29
(b) Udowodnić, że dla każdej dodatniej liczby całkowitej k wśród rozważanych liczb N istnieje liczba k-cyfrowa.
8. Dany jest graniastosłup prosty o wysokości 1, którego podstawa jest
ośmiokątem foremnym o boku 1, a punkty M1 , M2 , . . . , M10 są wszystkimi
środkami jego ścian. Dla dowolnego punktu P leżącego wewnątrz graniastosłupa niech Pi oznacza punkt przecięcia prostej Mi P z jego powierzchnią,
różny od Mi . Wybieramy punkt P w ten sposób, że żaden z punktów Pi nie
leży na krawędzi, a na każdej ścianie znajduje się dokładnie jeden z punktów Pi . Udowodnić, że
10
X
Mi P
i=1
M i Pi
= 5.
9. Niech n będzie liczbą całkowitą większą od 10 i niech A będzie zbiorem
2n-elementowym. Załóżmy, że A1 , A2 , . . . , Am są n-elementowymi podzbiorami zbioru A o tej własności, że każde przecięcie Ai ∩Aj ∩Ak dla i 6= j 6= k 6= i
ma co najwyżej 1 element. Znaleźć największą możliwą wartość m w zależności od n.
10. Dany jest ciąg a1 , a2 , . . . , a2010 . Suma każdych 20 kolejnych wyrazów tego ciągu jest nieujemna. Dla każdego i ∈ {1, 2, . . .P
, 2009} zachodzi
|ai − ai+1 | ¬ 1. Znaleźć najmniejszą możliwą wartość liczby 2010
i=1 ai .
XII Zawody Matematyczne Państw Bałtyckich
1. Na egzamin przygotowano 8 pytań. Każdy uczeń otrzymał 3 z nich.
Żadnych dwóch uczniów nie otrzymało więcej niż jedno wspólne pytanie. Jaka
największa możliwa liczba uczniów wzięła udział w egzaminie?
2. Niech n ­ 2 będzie liczbą całkowitą dodatnią. Rozstrzygnąć, czy istnieje n takich niepustych parami rozłącznych podzbiorów zbioru {1, 2, 3, . . .},
że każdą dodatnią liczbę całkowitą można w jednoznaczny sposób przedstawić w postaci sumy co najwyżej n liczb, każdej z innego podzbioru.
3. Liczby 1, 2, . . . , 49 rozmieszczono w tablicy 7 × 7, po czym obliczono
sumę liczb w każdym wierszu i każdej kolumnie. Niektóre z tych 14 sum
są nieparzyste, a pozostałe są parzyste. Niech A oznacza sumę wszystkich
nieparzystych sum, a B sumę wszystkich parzystych sum. Czy jest możliwe
takie rozmieszczenie liczb, że A = B?
4. Niech p i q będą dwiema różnymi liczbami pierwszymi. Dowieść, że
p
2p
3p
(q − 1)p
1
+
+
+...+
= (p − 1)(q − 1) .
q
q
q
q
2
([x] oznacza największą liczbę całkowitą nie większą niż x).
30
5. Każdy spośród danych 2001 punktów na okręgu pokolorowano na
czerwono albo na zielono. W jednym kroku wszystkie punkty są jednocześnie przekolorowywane w następujący sposób: jeśli oba punkty bezpośrednio
sąsiadujące z punktem P mają ten sam kolor co punkt P , to punkt P nie
zmienia koloru, w przeciwnym razie punkt P zmienia kolor. Rozpoczynając
od pokolorowania F1 otrzymujemy kolejno pokolorowania F2 , F3 , . . . . Udowodnić, że istnieje taka liczba n0 ¬ 1000, że Fn0 = Fn0 +2 . Czy to stwierdzenie
jest prawdziwe, jeśli liczbę 1000 zastąpimy przez 999?
6. Punkty A, B, C, D, E leżą w tej właśnie kolejności na okręgu c
i spełniają AB k EC oraz AC k ED. Prosta styczna do okręgu c w punkcie E
przecina prostą AB w punkcie P . Proste BD i EC przecinają się w punkcie Q.
Udowodnić, że AC = P Q.
7. Dany jest równoległobok ABCD. Okrąg przechodzący przez punkt A
przecina odcinki AB, AC i AD odpowiednio w punktach M , K i N , różnych
od A. Dowieść, że AB · AM + AD · AN = AK · AC .
8. Niech ABCD będzie czworokątem wypukłym i niech N będzie środ) AN D = 135◦ . Dowieść, że
kiem boku BC. Ponadto załóżmy, że <
BC
AB + CD + √ ­ AD .
2
9. Dany jest romb ABCD. Znaleźć zbiór takich punktów P , które leżą
) AP D + <
) BP C = 180◦ .
wewnątrz rombu i spełniają warunek <
10. W trójkącie ABC dwusieczna kąta BAC przecina bok BC w punk) ADB =45◦ , wyznaczyć miary kątów
cie D. Wiedząc, że BD·CD =AD2 oraz <
trójkąta ABC.
11. Funkcja f o wartościach rzeczywistych jest określona na zbiorze
wszystkich liczb całkowitych dodatnich. Dla dowolnych liczb całkowitych
a > 1, b > 1 oraz d = NWD(a, b) zachodzi równość
a
b
f (ab) = f (d) f
+f
.
d
d
Wyznaczyć wszystkie możliwe wartości f (2001).
12. Niech a1 , a2 , . . . , an będą takimi dodatnimi liczbami rzeczywistymi,
że zachodzą równości
n
X
i=1
Dowieść, że
n
X
a3i
=3
oraz
n
X
a5i = 5.
i=1
ai > 3/2.
i=1
13. Niech a0 , a1 , a2 , . . . będzie ciągiem liczb rzeczywistych spełniającym
warunki
a0 = 1
oraz
an = a[7n/9] + a[n/9]
dla n = 1, 2, 3, . . . .
31
Dowieść, że istnieje taka liczba całkowita dodatnia k, że
k
ak <
2001!
([x] oznacza największą liczbę całkowitą nie większą niż x).
14. Danych jest 2n kart. Na każdej karcie napisano pewną liczbę rzeczywistą x, przy czym 1 ¬ x ¬ 2 (na różnych kartach mogą być różne liczby).
Udowodnić, że można tak rozdzielić karty na dwa stosy o sumach odpowiednio s1 i s2 , że
s1
n
¬ ¬ 1.
n + 1 s2
15. Niech a0 , a1 , a2 , . . . będzie ciągiem dodatnich liczb rzeczywistych
spełniającym warunek
i · a2i ­ (i + 1) · ai−1 ai+1
dla
i = 1, 2, . . . .
Ponadto niech x i y będą dodatnimi liczbami rzeczywistymi oraz określmy
bi = xai + yai−1 dla i = 1, 2, . . .. Dowieść, że nierówność
i · b2i > (i + 1) · bi−1 bi+1
zachodzi dla wszystkich liczb całkowitych i ­ 2.
16. Niech f będzie funkcją o wartościach rzeczywistych określoną na
zbiorze liczb całkowitych dodatnich spełniającą następujący warunek: dla
wszystkich n > 1 istnieje taki dzielnik pierwszy p liczby n, że f (n) = f (n/p)−
f (p). Wiedząc, że f (2001) = 1, wyznaczyć f (2002).
17. Niech n będzie liczbą całkowitą dodatnią. Dowieść, że można wybrać
co najmniej 2n−1 + n takich liczb ze zbioru {1, 2, 3, . . . , 2n }, że dla dowolnych
wybranych dwóch różnych liczb x i y, x + y nie dzieli x · y.
n
n
18. Niech a będzie liczbą nieparzystą. Udowodnić, że liczby a2 + 22
m
m
i a2 +22 są względnie pierwsze dla dowolnych dodatnich liczb całkowitych
n i m, przy czym n 6= m.
19. Jaka jest najmniejsza dodatnia liczba nieparzysta mająca tyle samo
dodatnich dzielników co 360?
20. Z ciągu (a, b, c, d) liczb całkowitych możemy uzyskać w jednym kroku
jeden z ciągów
(c, d, a, b),
(b, a, d, c),
(a + nc, b + nd, c, d),
(a + nb, b, c + nd, d)
dla dowolnej liczby całkowitej n, przy czym liczba n może być w każdym
kroku inna. Rozstrzygnąć, czy przy pomocy pewnej liczby kroków można
otrzymać ciąg (3, 4, 5, 7) z ciągu (1, 2, 3, 4).
32
ROZWIĄZANIA ZADAŃ *
Zawody stopnia pierwszego
Zadanie 1.
Rozwiązać w liczbach rzeczywistych równanie
|x| − |x + 2| + |x + 4| − |x + 6| + . . . − |x + 998| =
= |x + 1| − |x + 3| + |x + 5| − |x + 7| + . . . − |x + 999| .
Rozwiązanie
Po przeniesieniu wszystkich wyrazów na lewą stronę zapisujemy równanie w postaci
f (x) + f (x + 4) + f (x + 8) + . . . + f (x + 996) = 0 ,
gdzie f (x) = |x| − |x + 1| − |x + 2| + |x + 3|.
Zauważmy, że f (x) > 0 dla −3 < x < 0 oraz f (x) = 0 dla x 6∈ (−3, 0). Zatem
liczba x spełnia dane równanie jedynie wtedy, gdy
f (x) = 0 ,
f (x + 4) = 0 ,
f (x + 8) = 0 ,
... ,
f (x + 996) = 0 ,
czyli wtedy i tylko wtedy, gdy
x 6∈ (−3, 0) ,
x 6∈ (−7, −4) ,
x 6∈ (−11, −8) ,
... ,
x 6∈ (−999, −996) .
Stąd wnioskujemy, że poszukiwanym zbiorem rozwiązań jest
(−∞, −999i ∪ h−996, −995i ∪ h−992, −991i ∪ . . . ∪ h−4, −3i ∪ h0, ∞) .
Zadanie 2.
Na bokach AB i AC trójkąta ABC zbudowano, po jego zewnętrznej
stronie, kwadraty ABDE i ACF G. Punkty M i N są odpowiednio
środkami odcinków DG i EF . Wyznaczyć możliwe wartości wyrażenia M N : BC.
Rozwiązanie
Sposób I
Niech X będzie takim punktem, że czworokąt EAGX jest równoległobokiem (rys. 1). Wówczas czworokąt DBGX jest również równoległobokiem, gdyż odcinki DB i XG są równe i równoległe. Tak więc punkt
M – środek odcinka DG – jest również środkiem odcinka BX. Analogicznie wnioskujemy, że punkt N jest środkiem odcinka CX. Zatem na mocy
twierdzenia odwrotnego do twierdzenia Talesa uzyskujemy M N k BC. Stąd
M N : BC = XM : XB = 1 : 2.
* Opracowali Adam Osękowski, Waldemar Pompe i Jarosław Wróblewski
33
Sposób II
W rozwiązaniu tym użyjemy rachunku wektorowego. Wiadomo bowiem,
że dla dowolnych czterech punktów P , Q, R i S prawdziwa jest równość
−−→ 1 −−→ −→
XY = 2 (P Q + RS) ,
gdzie X i Y są odpowiednio środkami odcinków P R i QS (rys. 2). Zatem
−−→ 1 −−→ −−→
−−→ −→
−−→
M N = 2 (DE + GF ) = 12 (BA + AC) = 12 BC .
Stąd uzyskujemy M N : BC = 1 : 2.
X
G
Q
F
N
M
E
P
A
Y
X
D
B
R
C
rys. 1
S
rys. 2
Uwaga.
W powyższych rozwiązaniach nie korzystaliśmy z założenia, że czworokąty ABDE i ACF G są kwadratami, a jedynie z tego, że są one równoległobokami .
Zadanie 3.
Wykazać, że liczba
10
10
X
n=0
!
10
2 · 1010 n
5 jest podzielna przez 22·10 −1 .
2n
Rozwiązanie
Udowodnimy najpierw następujący lemat.
Lemat.
Dla dowolnej liczby całkowitej dodatniej n istnieją takie liczby całkowite
a i b, że liczba a + b jest parzysta oraz
√
√
(1 ± 5)n = 2n−1 (a ± b 5) .
Dowód lematu.
Indukcja. Dla n = 1 lemat jest oczywiście spełniony. Niech n będzie taką
liczbą, że teza lematu jest prawdziwa. Wówczas
√
√
√
√
(1 ± 5)n+1 = 2n−1 (a ± b 5)(1 ± 5) = 2n−1 (a + 5b ± (a + b) 5) =
√
= 2n (c ± d 5) ,
34
gdzie c = (a+5b)/2 oraz d = (a+b)/2. Na mocy założenia indukcyjnego liczby
a i b są tej samej parzystości, więc liczby c oraz d są całkowite. Z tego samego
powodu liczba c + d = a + 3b jest parzysta. To kończy dowód lematu.
Na mocy wzoru dwumianowego Newtona
√
2·1010
(1 ± 5)
=
10
2·10
X
n=0
=
10
10
X
√
2 · 1010
(± 5)n =
n
!
10
2 · 1010 n 10X−1 2 · 1010 n √
5 ±
5 5.
2n
2n + 1
n=0
!
n=0
!
Stąd oraz korzystając z lematu uzyskujemy
10
10
X
n=0
√
√
10
10
2 · 1010 n 1 (1 + 5)2·10 + (1 − 5)2·10
=
5 =
2n
2
√
√ 10
1 2·1010 −1
=
2
(a + b 5) + 22·10 −1 (a − b 5) =
2
!
= 22·10
10
−1
a,
co dowodzi, że dana liczba jest podzielna przez 22·10
Zadanie 4.
10
−1
.
Dowieść, że wykres wielomianu W (x) stopnia większego od 1 ma
oś symetrii wtedy i tylko wtedy, gdy istnieją takie wielomiany F (x)
i G(x), że W (x) = F (G(x)), przy czym G(x) jest stopnia 2.
Rozwiązanie
Każdy wielomian G(x) stopnia drugiego można zapisać w postaci
G(x) = (x − a)2 + b ,
skąd G(x) = G(2a − x) .
Jeśli więc W (x) = F (G(x)), to
W (x) = W (2a − x) ,
a zatem wykres wielomianu W (x) ma pionową oś symetrii x = a.
Załóżmy teraz, że W (x) jest wielomianem stopnia większego od 1, a jego
wykres ma oś symetrii.
Jeżeli oś symetrii jest pionowa, powiedzmy x = a, to wykres wielomianu
V (x) = W (x + a) ma oś symetrii
o równaniu x = 0. Wówczas
V (x) = V (−x),
skąd V (x) jest postaci F x2 , a to daje W (x) = F (x − a)2 .
Oczywiście oś symetrii wykresu funkcji niestałej nie może być pozioma.
Udowodnimy teraz, że wykres wielomianu stopnia większego niż 1 nie
może mieć ukośnej osi symetrii. Załóżmy nie wprost, że wykres wielomianu
W (x) stopnia n ­ 2 ma ukośną oś symetrii. Bez szkody dla ogólności rozumowania możemy założyć, że oś symetrii przechodzi przez początek układu
współrzędnych. Istotnie, jeśli prosta o równaniu y = ax + b jest osią symetrii
35
wykresu wielomianu W (x), to prosta y = ax jest osią symetrii wielomianu
W (x) − b.
Zgodnie ze wzorem ze str. 107 Tablic matematycznych, A. Cewe, H. Nahorskiej, I. Pancer , symetria względem prostej y = ax przeprowadza punkt
(x, y) na punkt (x0 , y 0 ), gdzie x0 = Ax + By oraz y 0 = Bx − Ay, gdzie
2a
1 − a2
oraz B =
.
2
1+a
1 + a2
Przy tym B 6= 0, gdyż a 6= 0.
Wobec tego dla każdej liczby rzeczywistej x biorąc y = W (x) otrzymujemy y 0 = W (x0 ), czyli
A=
Bx − AW (x) = W (Ax + BW (x)) .
Pozostaje zauważyć, że wielomian po lewej stronie powyższej równości ma
stopień n (lub 1, gdy A = 0), a wielomian po stronie prawej ma stopień n2 .
Zatem powyższa równość nie może mieć miejsca dla wszystkich liczb rzeczywistych x.
Zadanie 5.
Wykazać, że dla każdej liczby naturalnej k istnieje taka liczba naturalna m, że każda z liczb m, 2m, 3m, . . . , m2 w rozwinięciu dwójkowym ma dokładnie k jedynek.
Rozwiązanie
Niech m = 2k −1. Liczba m jest liczbą, której zapis dwójkowy składa się
z k jedynek. Wtedy dla n ¬ m mamy
nm = (n − 1)2k + (2k − 1) − (n − 1) .
Dla n > 1 liczba nm wygląda więc w układzie dwójkowym następująco: na
początku znajduje się zapis dwójkowy liczby n−1, a następnie k-cyfrowy
zapis dwójkowy (z ew. początkowymi zerami) liczby (2k −1)−(n−1). Zatem
liczba nm ma w układzie dwójkowym tyle jedynek, ile mają razem liczby
n−1 i (2k −1)−(n−1), a ponieważ ich suma jest równa 2k −1, więc na każdej
pozycji dokładnie jedna z tych liczb posiada jedynkę. Tak więc liczba jedynek
w zapisie dwójkowym liczby nm jest równa k.
Uwaga.
Czytelnik niepewnie posługujący się układem dwójkowym może rozważyć następującą wersję zadania dla układu dziesiątkowego:
Wykazać, że dla każdej liczby naturalnej k istnieje taka liczba naturalna m, że każda z liczb m, 2m, 3m, . . . , m2 w rozwinięciu dziesiętnym ma
sumę cyfr równą 9k.
W rozwiązaniu wystarczy wziąć m = 10k − 1 i zauważyć, że
nm = (n − 1)10k + (10k − 1) − (n − 1) .
36
Zadanie 6.
Okrąg dzieli każdy bok rombu na 3 odcinki. Malujemy otrzymane
odcinki kolejno na czerwono, zielono i biało, zaczynając od wierzchołka rombu i poruszając się po jego obwodzie w ustalonym kierunku. Wykazać, że suma długości odcinków czerwonych jest równa
sumie długości odcinków białych.
Rozwiązanie
Sposób I
Oznaczmy przez P1 , P2 , . . . , P8 kolejne punkty przecięcia danego okręgu
z bokami rombu (rys. 3). Należy wykazać, że
AP1 + BP3 + CP5 + DP7 = P2 B + P4 C + P6 D + P8 A .
Niech Q1 , Q3 , Q5 , Q7 będą odpowiednio środkami odcinków P1 P2 , P3 P4 ,
P5 P6 , P7 P8 . Niech ponadto O będzie środkiem danego okręgu. Wtedy otrzymujemy OQ1 ⊥ AB, OQ3 ⊥ BC, OQ5 ⊥ CD oraz OQ7 ⊥ DA. Zatem odcinki
Q1 Q5 oraz Q3 Q7 są wysokościami rombu, a równość, którą należy udowodnić,
jest równoważna równości (rys. 4):
AQ1 + BQ3 + CQ5 + DQ7 = Q1 B + Q3 C + Q5 D + Q7 A .
(1)
Aby dowieść zależności (1), wystarczy wykazać, że jej lewa strona jest równa
połowie obwodu rombu ABCD.
) BAD¬90◦ (rys. 4). Niech BK i DL
Przyjmijmy dla ustalenia uwagi, że <
będą wysokościami rombu. Wtedy AK = AL, co dowodzi, że lewa strona równości (1) wynosi AL+CD+DK = AB +CD. To kończy rozwiązanie zadania.
D
P6
P5
C
Q5
D
Q5
C
P4
P7
Q7
Q7
O
P3
Q1
A
Q3
Q3
P8
P1
P2
rys. 3
B
K
A L
Q1
B
rys. 4
Sposób II
Podobnie jak w powyższym rozwiązaniu, sprowadzamy dowodzoną równość do postaci (1). Zależność tę przepisujemy w postaci
(2)
(AQ1 − Q1 B) + (BQ3 − Q3 C) + (CQ5 − Q5 D) + (DQ7 − Q7 A) = 0 .
Ponieważ AQ1 + Q1 B = BQ3 + Q3 C = CQ5 + Q5 D = DQ7 + Q7 A = a, więc
mnożąc obie strony równości (2) przez a uzyskujemy
(3) (AQ21 − Q1 B 2 ) + (BQ23 − Q3 C 2 ) + (CQ25 − Q5 D2 ) + (DQ27 − Q7 A2 ) = 0 .
37
Wówczas, na mocy twierdzenia Pitagorasa, równość (3) przybiera postać
(OA2 − OB 2 ) + (OB 2 − OC 2 ) + (OC 2 − OD2 ) + (OD2 − OA2 ) = 0 ,
co jest prawdą.
Uwaga.
W treści zadania romb można zastąpić dowolnym wielokątem wypukłym,
w którym wszystkie boki są równej długości. Dowód w tym ogólniejszym
przypadku jest analogiczny do rozwiązania ze sposobu II.
Zadanie 7.
W grupie n ­ 3 osób każda ma parzystą (być może zerową) liczbę
znajomych. Dowieść, że istnieją trzy osoby mające tę samą liczbę
znajomych.
Uwaga: Zakładamy, że nikt nie zalicza siebie samego do grona swoich
znajomych, oraz że osoba A zna osobę B wtedy i tylko wtedy, gdy
B zna A.
Rozwiązanie
Przypuśćmy, że teza zadania jest fałszywa, to znaczy, że żadne trzy osoby
nie mają takiej samej liczby znajomych.
Jeśli liczba n jest parzysta, to każda osoba z rozpatrywanej grupy ma
wśród pozostałych 0, 2, 4, . . . , n−4 lub n−2 znajomych. Ponieważ liczb tych
jest n/2, więc każda z nich musi być liczbą znajomych dla dokładnie dwóch
osób. W szczególności pewne dwie osoby nie mają ani jednego znajomego.
Wobec tego nikt nie ma więcej niż n−3 znajomych, nie zalicza bowiem do
grona swoich znajomych tych dwóch osób ani siebie. To jednak wyklucza
istnienie dwóch osób mających n−2 znajomych.
Jeśli natomiast liczba n jest nieparzysta, to każda osoba ma wśród pozostałych 0, 2, 4, . . . , n−3 lub n−1 znajomych. Ponieważ liczb tych jest (n+1)/2,
więc jedna z nich musi być liczbą znajomych dla dokładnie jednej osoby,
a każda z pozostałych liczbą znajomych dla dokładnie dwóch osób. W szczególności co najmniej jedna osoba nie ma ani jednego znajomego. Wobec tego
nikt nie ma więcej niż n−2 znajomych, co stoi w sprzeczności z istnieniem
osoby mającej n−1 znajomych.
Uzyskana sprzeczność dowodzi, że istnieją trzy osoby mające tę samą
liczbę znajomych.
Zadanie 8.
Niech S(n) oznacza sumę cyfr liczby n. Dowieść, że dla dowolnej
liczby naturalnej n liczba S(2n2 + 3) nie jest kwadratem liczby całkowitej.
Rozwiązanie
W rozwiązaniu skorzystamy z faktu, że kwadrat liczby całkowitej nie
może dawać z dzielenia przez 3 reszty 2, jak również z dzielenia przez 9
reszty 3. Istotnie: dla dowolnej liczby całkowitej n mamy n ≡ 0, ±1 (mod 3),
38
skąd n2 ≡0, 1 (mod 3). Zatem dla dowolnej liczby całkowitej n, n2 6≡ 2 (mod 3).
Podobnie, dla dowolnej liczby całkowitej n mamy
n ≡ 0, ±1, ±2, ±3, ±4
(mod 9) ,
2
co daje n ≡ 0, 1, 4, 7 (mod 9). Stąd w szczególności n2 6≡ 3 (mod 9).
Jeżeli teraz liczba n nie jest podzielna przez 3, to liczba 2n2 +3, a co za
tym idzie także S(2n2 +3), daje przy dzieleniu przez 3 resztę 2. Zatem liczba
ta nie może być kwadratem liczby całkowitej.
Jeśli natomiast liczba n jest podzielna przez 3, to liczba 2n2 +3, a co za
tym idzie również S(2n2 +3), daje przy dzieleniu przez 9 resztę 3. Jak zauważyliśmy wyżej, liczba ta również nie może być kwadratem liczby całkowitej.
Zadanie 9.
Płaszczyzna przecina krawędzie boczne graniastosłupa prawidłowego
sześciokątnego tworząc w przekroju sześciokąt wypukły D1 D2 . . . D6 .
Niech di (1 ¬ i ¬ 6) będzie odległością punktu Di od płaszczyzny
ustalonej podstawy graniastosłupa. Dowieść, że
d21 + d23 + d25 = d22 + d24 + d26 .
Rozwiązanie
Niech A1 A2 . . . A6 będzie ustaloną podstawą graniastosłupa. Niech ponadto A będzie środkiem tej podstawy. Dalej niech D będzie punktem przecięcia płaszczyzny sześciokąta D1 D2 . . . D6 z odcinkiem łączącym środki A
i B podstaw graniastosłupa.
Punkt A jest środkiem każdego z odcinków A1 A4 , A2 A5 , A3 A6 , więc na
mocy twierdzenia Talesa punkt D jest środkiem każdego z odcinków D1 D4 ,
D2 D5 , D3 D6 . Stąd
2AD = A1 D1 + A4 D4 = A2 D2 + A5 D5 = A3 D3 + A6 D6 .
Podobnie, prosta A1 A4 jest środkową w trójkącie A1 A3 A5 , więc na mocy
twierdzenia Talesa prosta D1 D4 jest środkową w trójkącie D1 D3 D5 . Przeprowadzając analogiczne rozumowanie dla prostych D2 D5 , D3 D6 wnioskujemy,
że punkt D jest środkiem ciężkości trójkąta D1 D3 D5 . Ta sama argumentacja
prowadzi do wniosku, że punkt D jest środkiem ciężkości trójkąta D2 D4 D6 .
Stąd uzyskujemy
3AD = A1 D1 + A3 D3 + A5 D5 = A2 D2 + A4 D4 + A6 D6 .
Zatem
d21 − d22 + d23 − d24 + d25 − d26 =
= A1 D12 − A4 D42 + A5 D52 − A2 D22 + A3 D32 − A6 D62 =
= (A1 D1 + A4 D4 ) (A1 D1 − A4 D4 ) + (A5 D5 + A2 D2 ) (A5 D5 − A2 D2 ) +
+ (A3 D3 + A6 D6 ) (A3 D3 − A6 D6 ) =
= 2AC (A1 D1 − A4 D4 + A5 D5 − A2 D2 + A3 D3 − A6 D6 ) = 0 ,
czego należało dowieść.
39
Zadanie 10. Dana jest szachownica 2000 × 2000. Na każdym polu leży kamień.
Wykonujemy następujące ruchy: jeśli na pierwszym i trzecim z trzech
kolejnych pól leżących w jednym wierszu lub kolumnie leży kamień,
to możemy oba te kamienie przełożyć na drugie z tych pól (niezależnie od tego, czy jakiś kamień leży na środkowym polu, i czy ruch
opróżni jakiekolwiek pole).
Rozstrzygnąć, czy można wykonując takie ruchy przełożyć wszystkie
kamienie na jedno pole.
Rozwiązanie
Odp.: Nie można.
Przyjmijmy, że każde pole szachownicy jest kwadratem o boku 1. Umieśćmy daną szachownicę w układzie współrzędnych tak, aby środek każdego
pola pokrywał się z pewnym punktem kratowym o współrzędnych ze zbioru
{1, 2, . . . , 2000}. W ten sposób przypisujemy każdemu polu szachownicy parę
współrzędnych ze zbioru {1, 2, . . . , 2000}.
Kamieniom leżącym odpowiednio na polach o współrzędnych (x1 , y1 ),
(x2 , y2 ), . . . , (x20002 , y20002 ) przypisujemy parę liczb
x1 + x2 + . . . + x20002 y1 + y2 + . . . + y20002
S=
,
,
20002
20002
czyli środek ciężkości zbioru wszystkich kamieni. Bez trudu sprawdzamy, że
dane w zadaniu ruchy nie zmieniają położenia punktu S.
Zatem gdyby udało się przełożyć wszystkie kamienie na jedno pole, to
pole to musiałoby pokrywać się z punktem S odpowiadającym kamieniom
znajdującym się w położeniu wyjściowym, czyli mieć współrzędne
(1000,5 ; 1000,5) .
To jednak nie jest możliwe, gdyż współrzędne środków wszystkich pól szachownicy są liczbami ze zbioru {1, 2, . . . , 2000}.
Zadanie 11. Dany jest trójkąt ABC, w którym <) B > <) C. Punkt D leży na od) DAC = 12 (<
)B −<
) C) . Okrąg styczny
cinku BC i spełnia równość <
w punkcie A do prostej AC i przechodzący przez punkt D przecina
prostą AB w punkcie P 6= A. Dowieść, że
BD
BP
=
.
AC
DC
Rozwiązanie
Oznaczmy przez E drugi punkt przecięcia prostej BC z danym okręgiem
) ADB = <
) EP B oraz <
) ABD = <
) EBP wniosku(rys. 5). Z równości kątów <
jemy, że trójkąty ABD i EBP są podobne. Stąd
BP
EB
=
.
(1)
BD AB
) CAD = <
) CEA oraz <
) ACD = <
) ECA wynika, że trójkąty
Dalej, z równości <
40
ACD i ECA są podobne. To daje
AC EC
(2)
=
.
DC AC
Na mocy równości (1) i (2) przepisujemy dowodzoną równość w postaci
EB EC
AB EB
=
, czyli
=
.
AB AC
AC EC
Należy zatem wykazać, że AE jest dwusieczną kąta zewnętrznego przy wierzchołku A trójkąta ABC.
A
E
B
D
C
P
rys. 5
Korzystając z danej w treści zadania równości kątów otrzymujemy
<
) EAB = <
)B −<
) AEB = <
)B −<
) DAC = 21 (<
)B +<
) C) .
Równość ta oznacza właśnie, że prosta AE jest dwusieczną kąta zewnętrznego
przy wierzchołku A trójkąta ABC. To kończy rozwiązanie zadania.
Zadanie 12. W niemalejącym ciągu a1 , a2 , a3 , . . . wszystkie wyrazy są liczbami całkowitymi dodatnimi, a ponadto dla każdego k dokładnie k wyrazów
jest równych k. Wyznaczyć wszystkie liczby pierwsze postaci
a1 + a2 + . . . + an .
Rozwiązanie
Z podanych warunków wynika, że ciąg (an ) wygląda następująco:
1, 2, 2, 3, 3, 3, 4, 4, 4, 4, 5, 5, 5, 5, 5, . . . .
Dla dowolnej liczby naturalnej n ­ 1 istnieją takie całkowite 0 ¬ ` ¬ k, że
n = (1 + 2 + 3 + . . . + k) + ` .
Wówczas
sn = a1 + a2 + . . . + an = 1 · 1 + 2 · 2 + 3 · 3 + . . . + k · k + ` · (k + 1) =
k(k + 1)(2k + 1)
+ `(k + 1) .
6
Jeżeli liczba k+1 nie jest dzielnikiem liczby 6, tzn. jeśli k 6= 1, 2, 5, to powyższa
liczba jest złożona, gdyż jest podzielna przez d = (k + 1)/ NWD(k+1, 6) > 1,
a przy tym sn /d ­ k(2k + 1)/6 > 1.
=
41
Pozostają do rozpatrzenia przypadki k = 1, 2, 5, które bezpośrednio analizujemy:
k = 1,
k = 1,
k = 2,
k = 2,
k = 2,
k = 5,
k = 5,
k = 5,
k = 5,
k = 5,
k = 5,
` = 0:
` = 1:
` = 0:
` = 1:
` = 2:
` = 0:
` = 1:
` = 2:
` = 3:
` = 4:
` = 5:
sn = 1 ;
sn = 1 + 2 = 3 ;
sn = 1 + 2 + 2 = 5 ;
sn = 1 + 2 + 2 + 3 = 8 ;
sn = 1 + 2 + 2 + 3 + 3 = 11 ;
sn = 1 + 2 · 2 + 3 · 3 + 4 · 4 + 5 · 5 = 55 ;
sn = 1 + 2 · 2 + 3 · 3 + 4 · 4 + 5 · 5 + 1 · 6 = 61 ;
sn = 1 + 2 · 2 + 3 · 3 + 4 · 4 + 5 · 5 + 2 · 6 = 67 ;
sn = 1 + 2 · 2 + 3 · 3 + 4 · 4 + 5 · 5 + 3 · 6 = 73 ;
sn = 1 + 2 · 2 + 3 · 3 + 4 · 4 + 5 · 5 + 4 · 6 = 79 ;
sn = 1 + 2 · 2 + 3 · 3 + 4 · 4 + 5 · 5 + 5 · 6 = 85 .
Liczbami pierwszymi żądanej postaci są więc: 3, 5, 11, 61, 67, 73 i 79.
42
Zawody stopnia drugiego
Zadanie 1.
Dana jest taka funkcja f : R → R, że dla każdej liczby rzeczywistej x
zachodzą równości
f (x) = f (2x) = f (1 − x) .
Dowieść, że funkcja f jest okresowa.
Rozwiązanie
Na mocy danych w treści zadania równości, otrzymujemy dla dowolnej
liczby rzeczywistej x:
f (x) = f (2x) = f (1 − 2x) =
= f ( 21 (1 − 2x)) = f ( 12 − x) = f (1 − ( 12 − x)) = f (x + 21 ) ,
co dowodzi, że funkcja f jest okresowa o okresie 21 .
Zadanie 2.
W czworokącie wypukłym ABCD zachodzą równości
<
) ADB = 2<
) ACB oraz <
) BDC = 2<
) BAC .
Udowodnić, że AD = CD.
Rozwiązanie
Sposób I
) ADC < 180◦
Z danych w zadaniu równości kątów oraz z nierówności <
wynika, że
) ADB + <
) BDC) = 12 <
) ADC < 90◦ .
<
) ACB + <
) BAC = 21 (<
Kąty ACB i BAC są więc ostre, co oznacza, że środek O okręgu opisanego
na trójkącie ABC leży wewnątrz kąta ABC (rys. 6). Stąd w szczególności
wynika, że punkty O i D leżą po tej samej stronie prostej BC. Ponadto
<
) BDC = 2<
) BAC = <
) BOC .
Zatem punkty B, C, O, D leżą na jednym okręgu o1 .
B
C
A
OD
rys. 6
Analogicznie dowodzimy, że punkty A, B, O oraz D leżą na jednym
okręgu o2 . Okręgi o1 i o2 są różne — w przeciwnym razie okrąg opisany na
trójkącie ABC przechodziłby przez punkt O, czyli swój środek. Okręgi o1
43
i o2 mają jednak trzy punkty wspólne: B, D i O. Ponieważ B 6= D, więc musi
być O = D, czyli AD = CD.
Sposób II
Tak jak w sposobie I dowodzimy, że
<
) BAC + <
) ACB < 90◦ .
(1)
) DBA > <
) BAC lub <
) DBC > <
) ACB — w przeciwnym razie mieTo daje <
) ABC = <
) DBA + <
) DBC ¬ <
) BAC + <
) ACB < 90◦ . Stąd, na mocy
libyśmy <
nierówności (1), suma kątów w trójkącie ABC byłaby mniejsza niż 180◦ . Bez
) DBA > <
) BAC.
straty ogólności przyjmijmy więc, że <
E
B
C
A
D
rys. 7
Prostą DB przedłużamy do takiego punktu E, że zachodzi równość
<
) BEA = <
) DBA − <
) BAC (rys. 7). Wówczas
<
) BAE = <
) DBA − <
) BEA = <
) BAC oraz <
) EDC = 2<
) BAC = <
) EAC ,
co oznacza, że na czworokącie CDAE można opisać okrąg. Stąd uzyskujemy
) BCE = <
) BCA.
<
) ADE = <
) ACE, co daje <
) ACB = 21 <
) ACE, czyli <
)A i <
)C
Punkt B jest więc punktem przecięcia dwusiecznych kątów <
w trójkącie ECA. Stąd wynika, że EB jest dwusieczną kąta CEA, a to oznacza, że AD = CD.
Uwaga.
Z danych w treści zadania równości kątów wynika nie tylko, że AD =CD,
lecz również AD = CD = BD.
Zadanie 3.
W n-osobowym stowarzyszeniu działa sześć komisji. W skład każdej
z nich wchodzi nie mniej niż n/4 osób. Dowieść, że istnieją dwie
komisje oraz grupa licząca nie mniej niż n/30 osób, należących do
obu tych komisji.
Rozwiązanie
Ponumerujmy komisje liczbami 1, 2, . . . , 6 i oznaczmy przez Ki liczbę tych
członków i-tej komisji, którzy nie są członkami żadnej komisji o numerze
mniejszym od i.
44
K1 to liczba wszystkich członków komisji nr 1, co daje K1 ­ n/4.
Przypuśćmy, że część wspólna dowolnych dwóch komisji liczy mniej niż
n/30 osób.
Ponieważ w skład komisji nr 2 wchodzi co najmniej n/4 osób, więc liczba
tych członków komisji nr 2, którzy nie są w komisji nr 1 musi być większa od
n/4 − n/30. Stąd
n n
K2 > − .
4 30
Analogicznie stwierdzamy, że
n
n
Ki > − (i − 1)
dla i = 3, 4, 5, 6.
4
30
Dodając stronami powyższe nierówności uzyskujemy
6n
n
− (1 + 2 + 3 + 4 + 5)
=n.
K1 + K2 + K3 + K4 + K5 + K6 >
4
30
Otrzymaliśmy sprzeczność, gdyż wielkość K1 + K2 + . . . + K6 jest nie większa od liczby wszystkich członków stowarzyszenia, czyli n. Zatem do części
wspólnej pewnych dwóch komisji należy co najmniej n/30 osób.
Zadanie 4.
Wyznaczyć wszystkie takie trójki liczb pierwszych p ¬ q ¬ r, że liczby
pq + r,
pq + r2 ,
qr + p,
qr + p2 ,
rp + q,
rp + q 2
są pierwsze.
Rozwiązanie
Gdyby wszystkie trzy liczby pierwsze p, q, r były większe od 2, to liczba
pq + r jako liczba parzysta większa od 2 byłaby złożona. Zatem co najmniej
jedna z liczb p, q, r jest równa 2, co na mocy nierówności p ¬ q ¬ r daje p = 2.
Ponadto q > 2, gdyż przy q = 2 mielibyśmy qr+p = 2(r+1), co jest liczbą
złożoną.
Gdyby obie liczby pierwsze q, r były większe od 3, to liczba qr byłaby
niepodzielna przez 3, a zatem jedna z liczb qr + 2, qr + 4 byłaby podzielna
przez 3 i większa od 3, czyli złożona. Stąd wynika, że jedna z liczb q, r jest
równa 3, co na mocy nierówności 2 < q ¬ r daje q = 3.
Pozostało wyznaczyć wszystkie takie liczby pierwsze r ­ 3, aby liczby
(1)
6 + r,
6 + r2 ,
3r + 2,
3r + 4,
2r + 3,
2r + 9
były pierwsze.
Gdyby liczba pierwsza r była różna od 5, to przez 5 dzieliłaby się jedna
z liczb r + 1, r + 2, r + 3, r + 4, a co za tym idzie, również jedna z liczb
6 + r = (r + 1) + 5 ,
2r + 9 = 2(r + 2) + 5 ,
3r + 4 = 3(r + 3) − 5 ,
2r + 3 = 2(r + 4) − 5 .
Zatem musi być r = 5. Wtedy liczby (1) przyjmują odpowiednio wartości 11,
31, 17, 19, 13, 19 i wszystkie one są pierwsze.
Odp.: Warunki zadania spełnia jedna trójka (p, q, r) = (2, 3, 5).
45
Zadanie 5.
) BAC = 90◦ , jest podstawą ostrosłupa
Trójkąt ABC, w którym <
ABCD. Ponadto zachodzą równości
AD = BD
AB = CD .
oraz
◦
) ACD ­ 30 .
Udowodnić, że <
Rozwiązanie
Trójkąt BAC uzupełniamy do prostokąta BACE. Wówczas z równości
AD = BD wynika, że CD = ED. To w połączeniu z AB = CD dowodzi, że
trójkąt CDE jest równoboczny. Zatem
<
) ACD ­ <
) ACE − <
) DCE = 90◦ − 60◦ = 30◦ .
Uwaga.
) ACD =30◦ zachodzi wtedy i tylko wtedy, gdy punkt D należy
Równość <
do płaszczyzny trójkąta ABC.
Zadanie 6.
Wyznaczyć wszystkie takie liczby naturalne n, że dla dowolnych liczb
rzeczywistych x1 , x2 , . . . , xn , y1 , y2 , . . . , yn zachodzi nierówność
(1)
x1 x2 . . . xn + y1 y2 . . . yn ¬
q
x21 + y12 ·
q
x22 + y22 · . . . ·
q
x2n + yn2 .
Rozwiązanie
Sposób I
Dla n = 1 dana nierówność przybiera postać
x1 + y1 ¬
q
x21 + y12 ,
co nie dla wszystkich liczb rzeczywistych x1 , y1 jest prawdą — wystarczy
przyjąć x1 = y1 = 1.
Wykażemy, że nierówność (1) jest prawdziwa, jeśli n ­ 2.
Jeśli któryś z czynników stojących po prawej stronie nierówności (1), np.
q
x2k + yk2
jest równy 0, to obie liczby xk i yk muszą być zerami. Wtedy nierówność (1)
jest spełniona.
Przyjmijmy więc, że liczba stojąca po prawej stronie nierówności (1)
jest różna od 0, a zatem dodatnia. Wówczas dowodzoną nierówność możemy
sprowadzić do postaci
x1
xn
y1
yn
q
(2)
·...· q
+q
·...· q
¬ 1.
x21 + y12
x2n + yn2
x21 + y12
x2n + yn2
Podstawmy:
ak =
46
x2k
,
x2k + yk2
bk =
yk2
x2k + yk2
(k = 1, 2, . . . , n) .
Liczby a1 , a2 , . . . , an , b1 , b2 , . . . , bn należą do przedziału h0, 1i oraz ak + bk = 1.
Stąd oraz z nierówności pomiędzy średnią geometryczną a średnią arytmetyczną uzyskujemy
p
a1 a2 . . . an +
p
b1 b2 . . . bn ¬
p
n
p
n
b1 b2 . . . bn ¬
1
1
¬ (a1 + a2 + . . . + an ) + (b1 + b2 + . . . + bn ) = 1 ,
n
n
a1 a2 . . . an +
czyli
(3)
|x1 |
|yn |
|xn |
|y1 |
¬ 1.
·...· q
+q
·...· q
x2n + yn2
x21 + y12
x2n + yn2
x21 + y12
q
Lewa strona nierówności (2) jest nie większa niż lewa strona nierówności (3).
To oznacza, że udowodniona właśnie nierówność (3) pociąga za sobą nierówność (2), a tym samym dowodzi nierówności (1).
Sposób II
Podobnie jak w sposobie I stwierdzamy, że nierówność (1) nie jest prawdziwa dla n = 1.
Załóżmy więc, że n ­ 2. Bez szkody dla ogólności możemy przyjąć, że
liczby x1 , x2 , . . . , xn , y1 , y2 , . . . , yn są nieujemne.
Podnosząc nierówność (1) stronami do kwadratu, sprowadzamy ją do
postaci
x21 x22 . . . x2n + y12 y22 . . . yn2 + 2x1 x2 . . . xn y1 y2 . . . yn ¬
¬ (x21 + y12 )(x22 + y22 ) . . . (x2n + yn2 ) .
Po wymnożeniu prawa strona powyższej nierówności zawiera 2n iloczynów,
z których każdy jest nieujemny. Cztery z tych iloczynów to
x21 x22 . . . x2n ,
y12 y22 . . . yn2 ,
x21 y22 y32 . . . yn2 ,
y12 x22 x23 . . . x2n .
Stąd uzyskujemy nierówność
x21 x22 . . . x2n + y12 y22 . . . yn2 + x21 y22 . . . yn2 + y12 x22 . . . x2n ¬
¬ (x21 + y12 )(x22 + y22 ) . . . (x2n + yn2 ) .
Zatem zadanie będzie rozwiązane, jeśli wykażemy, że
x21 x22 . . . x2n + y12 y22 . . . yn2 + 2x1 x2 . . . xn y1 y2 . . . yn ¬
¬ x21 x22 . . . x2n + y12 y22 . . . yn2 + x21 y22 . . . yn2 + y12 x22 . . . x2n .
Jednak powyższa nierówność jest równoważna nierówności
(x1 y2 y3 . . . yn − y1 x2 x3 . . . xn )2 ­ 0 ,
która jest prawdziwa.
47
Zawody stopnia trzeciego
Zadanie 1.
Wyznaczyć wszystkie takie trójki liczb naturalnych a, b, c, że liczby
a2 + 1 i b2 + 1 są pierwsze oraz
(a2 + 1)(b2 + 1) = c2 + 1 .
Rozwiązanie
Bez szkody dla ogólności rozwiązania możemy założyć, że a ¬ b. Niech
p = b2 + 1. Wówczas liczba (c2 + 1) − (b2 + 1) = (c − b)(c + b) dzieli się przez p.
Z danej w treści zadania równości wynika, że b < c. Ponadto
c=
q
(a2 + 1)(b2 + 1) − 1 ¬
q
(b2 + 1)2 − 1 < b2 + 1 = p .
Stąd c − b < p oraz c + b < 2p. Skoro p | (c − b)(c + b), więc musi być c + b = p,
czyli
(1)
c = b(b − 1) + 1 .
Liczby a = b = 1 nie spełniają warunków zadania; możemy więc przyjąć,
że b > 1. Wówczas p jest liczbą pierwszą nieparzystą. Z zależności (1) wynika,
że c jest nieparzyste, a to dowodzi kolejno parzystości liczb c2 +1 oraz a2 +1.
Liczba a2 + 1 jest pierwsza, a więc a2 + 1 = 2, skąd a = 1. Zatem dane w zadaniu równanie przybiera postać
(2)
2(b2 + 1) = c2 + 1 .
Łącząc równości (1) i (2) uzyskujemy 2b2 + 2 = b2 (b − 1)2 + 2b(b − 1) + 2 ,
czyli 2b = b(b−1)2 +2b−2 . Stąd b jest dzielnikiem liczby 2, a ponieważ b > 1,
więc musi być b = 2. To pociąga za sobą c = 3.
Pozostaje stwierdzić, że trójka (a, b, c)=(1, 2, 3) spełnia warunki zadania.
Uwalniając się od założenia a ¬ b otrzymujemy dwie trójki (a, b, c) spełniające warunki zadania. Są nimi: (1, 2, 3) oraz (2, 1, 3).
Zadanie 2.
Na bokach AC i BC trójkąta ostrokątnego ABC zbudowano, po jego
zewnętrznej stronie, prostokąty ACP Q i BKLC o równych polach.
Udowodnić, że środek odcinka P L, punkt C oraz środek okręgu opisanego na trójkącie ABC leżą na jednej prostej.
Rozwiązanie
Oznaczmy przez O środek okręgu opisanego na trójkącie ABC (rys. 8).
Uzupełnijmy trójkąt P CL do równoległoboku P CLX. Wystarczy udowodnić, że punkty X, C, O są współliniowe.
Z równości pól danych prostokątów otrzymujemy
XL P C BC
=
=
.
LC
LC
CA
) XLC = 180◦ − <
) P CL = <
) BCA. Uzyskane zależności dowodzą, że
Ponadto <
trójkąty XLC oraz BCA są podobne. Stąd oraz z równości BO = CO wynika
48
kolejno, że
<
) XCL = <
) BAC = 12 <
) BOC = 90◦ − <
) BCO ,
) XCL+ <
) LCB + <
) BCO = 180◦ . Ostatnia równość oznacza, że punkty
czyli <
X, C, O leżą na jednej prostej.
X
L
P
C
K
O
Q
A
B
rys. 8
Zadanie 3.
Na tablicy są napisane trzy nieujemne liczby całkowite. Wybieramy
z tej trójki dwie liczby k, m i zastępujemy je liczbami k+m i |k−m|,
a trzecia liczba pozostaje bez zmiany. Z otrzymaną trójką postępujemy tak samo. Rozstrzygnąć, czy z każdej początkowej trójki
liczb całkowitych nieujemnych, kontynuując to postępowanie, można
otrzymać trójkę, w której co najmniej dwie liczby są zerami.
Rozwiązanie
Każdą trójkę liczb całkowitych nieujemnych można przedstawić w postaci
(1)
(2p a, 2q b, 2r c)
gdzie p, q, r są liczbami całkowitymi nieujemnymi, zaś każda z liczb a, b, c jest
nieparzysta lub równa 0. Wagą trójki przedstawionej w postaci (1) nazwiemy
wielkość a + b + c.
Wykażemy, że z każdej trójki liczb całkowitych nieujemnych o co najmniej dwóch liczbach niezerowych, wykonując operacje opisane w treści zadania, da się uzyskać trójkę o mniejszej wadze. Tym samym udowodnimy, że
zawsze można otrzymać trójkę, w której co najmniej dwie liczby są zerami.
Załóżmy więc, że w trójce (2p a, 2q b, 2r c) co najmniej dwie liczby są różne
od 0. Bez straty ogólności przyjmijmy, że b, c 6= 0 oraz q ¬ r.
Wykonując 2(r−q)-krotnie operację (k, l, m) 7→ (k + l, |k − l|, m) przeprowadzamy trójkę (2p a, 2q b, 2r c) na trójkę (2p+r−q a, 2r b, 2r c), którą następnie
przekształcamy na (2p+r−q a, 2r |b − c|, 2r (b + c)) . Liczby b i c są nieparzyste,
49
więc waga ostatniej trójki nie przekracza
a + 12 |b − c| + 12 (b + c) = a + max(b, c) ,
co jest mniejsze od a + b + c, czyli od wagi trójki (2p a, 2q b, 2r c).
Zadanie 4.
Dowieść, że dla każdej liczby naturalnej n­3 i dla każdego ciągu liczb
dodatnich x1 , x2 , . . . , xn zachodzi co najmniej jedna z nierówności
n
X
n
X
xi
n
­ ,
x
+
x
2
i+2
i=1 i+1
xi
n
­
x
+
x
2
i−2
i=1 i−1
(przyjmujemy xn+1 = x1 , xn+2 = x2 oraz x0 = xn , x−1 = xn−1 ).
Rozwiązanie
Gdyby żadna z podanych nierówności nie zachodziła, to po dodaniu
n
X
xi−1 + xi+2
< n . Wystarczy zatem udowodnić, że
stronami mielibyśmy
xi + xi+1
i=1
n
X
xi−1 + xi+2
i=1
xi + xi+1
­ n,
albo, że
n
X
xi−1 + xi + xi+1 + xi+2
xi + xi+1
i=1
Po podstawieniu ai =
n X
ai +
i=1
xi−1 + xi
ostatnia nierówność przybiera postać
xi + xi+1
1
ai+1
a to jest prawda, gdyż a +
Zadanie 5.
­ 2n .
­ 2n ,
czyli
n X
ai +
i=1
1
ai
­ 2n ,
1
­ 2 dla dowolnej liczby dodatniej a.
a
W przestrzeni dany jest trójkąt ABC oraz sfera s rozłączna z płaszczyzną ABC. Przez każdy z punktów A, B, C poprowadzono prostą styczną do tej sfery. Punkty styczności oznaczono odpowiednio
K, L, M . Punkt P leży na sferze s i spełnia warunki
BL CM
AK
=
=
.
AP
BP
CP
Udowodnić, że sfera opisana na czworościanie ABCP jest styczna do
sfery s.
Rozwiązanie
Oznaczmy przez λ wspólną wartość danych w treści zadania ułamków.
Niech A0 , B 0 , C 0 będą odpowiednio drugimi punktami przecięcia prostych
P A, P B, P C ze sferą s (rys. 9 i 10). Jeśli któraś z tych prostych, powiedzmy
P A, jest styczna do sfery s, to przyjmujemy A0 =P . Wówczas AK 2 =AP ·AA0 ,
50
a więc
AA0
AK
=
AP
AP
2
= λ2 .
Zatem
AA0 BB 0 CC 0
=
=
= λ2 .
AP
BP
CP
Stąd wynika, że jednokładność j o środku P przekształcająca punkt A na A0 ,
przeprowadza punkt B na B 0 oraz punkt C na C 0 . Ponadto każdy z punktów
A0 , B 0 , C 0 jest różny od P . W przeciwnym razie równości (1) implikują,
że λ = 1, czyli że proste P A, P B, P C są styczne do sfery s w punkcie P .
Wtedy P byłby punktem wspólnym płaszczyzny ABC i sfery s, co przeczy
założeniom.
(1)
s
s
P
B0
0
A
P
C0
B0
A0
C
C
0
B
A
A
B
rys. 9
C
rys. 10
Sfera opisana na czworościanie ABCP jest zatem obrazem sfery s przy
jednokładności j −1 , której środek (punkt P ) leży na sferze s. Stąd teza.
Zadanie 6.
Dana jest liczba naturalna k. Określamy ciąg (an ) wzorami
a1 = k + 1,
an+1 = a2n − kan + k dla n ­ 1 .
Wykazać, że jeżeli m 6= n, to liczby am i an są względnie pierwsze.
Rozwiązanie
Udowodnimy najpierw indukcyjnie, że dla n ­ 1 zachodzi równość
(1)
an+1 = a1 a2 . . . an + k .
Z danej w treści zadania zależności rekurencyjnej obliczamy
a2 = (k + 1)2 − k(k + 1) + k = 2k + 1 = a1 + k ,
51
co dowodzi prawdziwości zdania (1) dla n = 1, natomiast krok indukcyjny
sprowadza się do ciągu równości:
an+1 = a2n − kan + k = an (a1 a2 . . . an−1 + k) − kan + k = a1 a2 . . . an + k .
Przypuśćmy teraz, że p jest wspólnym dzielnikiem pierwszym liczb al
i am , przy czym l > m. Wówczas z zależności (1) dla n = l−1 wynika, że p | k,
a więc liczba a1 nie jest podzielna przez p. Niech r będzie najmniejszą liczbą
naturalną, dla której p | ar (taka liczba r istnieje). Wtedy 1 < r ¬ m. Stosując
ponownie równość (1), lecz tym razem dla n = r−1 wnioskujemy, że p jest
dzielnikiem jednej z liczb a1 , a2 , . . . , ar−1 , wbrew temu, że r jest najmniejszą
liczbą o własności p | ar .
Otrzymana sprzeczność dowodzi, że liczby al i am są względnie pierwsze.
52
XLII Międzynarodowa Olimpiada Matematyczna
Zadanie 1.
W trójkącie ostrokątnym ABC punkt O jest środkiem okręgu opisanego. Punkt P leżący na boku BC jest spodkiem wysokości z wierz) BCA ­ <
) ABC + 30◦ . Wykazać, że
chołka A. Zakładamy, że <
<
) CAB + <
) COP < 90◦ .
Rozwiązanie
Sposób I
Odbijmy punkty A i P symetrycznie względem symetralnej odcinka BC
i otrzymane punkty nazwijmy odpowiednio K i Q (rys. 11). Niech R będzie
długością promienia okręgu opisanego na trójkącie ABC. Zauważmy, iż na
) BCA > <
) ABC, skąd
mocy warunków zadania, <
<
) AOK = <
) AOB − <
) KOB = <
) AOB − <
) AOC = 2(<
) BCA − <
) ABC) ­ 60◦ .
K
A
O
B
Q
P
C
rys. 11
Ponieważ OA = OK = R, więc AK ­ R. Ale P Q = AK, gdyż czworokąt
P AKQ jest prostokątem. Stąd P Q ­ R i korzystając z nierówności trójkąta,
mamy OP +R = OQ+OC > QC = QP +P C ­ R+P C. Zatem OP > P C, skąd
) P CO > <
) COP . Wystarczy już tylko zauważyć, że
otrzymujemy <
1
1
<
) CAB = <
) BOC = (180◦ − 2<
) P CO) = 90◦ − <
) P CO,
2
2
) CAB + <
) COP < 90◦ .
skąd otrzymujemy tezę: <
Sposób II
Niech R będzie promieniem okręgu opisanego na trójkącie ABC.
) ABC oraz γ = <
) BCA. Na mocy twierdzenia siOznaczmy ponadto: β = <
nusów uzyskujemy AB = 2R sin γ oraz AC = 2R sin β. Stąd otrzymujemy
BP − P C = AB cos β − AC cos γ = 2R(sin γ cos β − sin β cos γ) = 2R sin(γ − β),
co w połączeniu z nierównościami
30◦ ¬ γ − β < γ < 90◦
daje BP −P C ­R. Zatem R+OP =BO+OP >BP ­R+P C, czyli OP >P C.
Stąd wynika teza zadania (jak w pierwszym rozwiązaniu).
53
Sposób III
Wprowadźmy oznaczenia takie jak w sposobie II. Dodatkowo oznaczmy:
) CAB oraz δ = <
) COP . Udowodnimy najpierw, że R2 > CP ·CB. Ponieα=<
waż CB = 2R sin α oraz CP = AC cos γ = 2R sin β cos γ, więc wystarczy wykazać, że 41 > sin α sin β cos γ. Ale mamy
1 > sin α = sin(γ + β) = sin γ cos β + sin β cos γ
oraz, ze względu na nierówność 30◦ ¬ γ − β < 90◦ ,
1
¬ sin(γ − β) = sin γ cos β − sin β cos γ.
2
Stąd 14 > sin β cos γ, a więc tym bardziej 14 > sin α sin β cos γ.
A
O
J
B
P
C
rys. 12
Na prostej CB → wybierzmy taki punkt J, aby CJ · CP = R2 (rys. 12).
Ponieważ, jak wykazaliśmy wyżej, R2 > CP · CB, więc CJ > CB, a więc
<
) OBC > <
) OJC. Ale mamy
OC P C
=
CJ
OC
<
)
<
)
oraz JCO = OCP . Stąd wynika, że trójkąty JCO i OCP są podobne oraz
<
) OJC = <
) P OC = δ. Zatem δ < <
) OBC = 90◦ − α, czyli α + δ < 90◦ .
Sposób IV
Tak jak w trzecim rozwiązaniu dowodzimy, że R2 > CP · CB. Z drugiej
strony, potęga punktu P względem okręgu opisanego na trójkącie ABC wynosi −BP · P C = OP 2 − R2 . Stąd otrzymujemy
OP 2 = R2 − BP · P C > P C · CB − BP · P C = P C 2 ,
co daje OP > P C. Ta nierówność implikuje tezę zadania, jak w pierwszym
rozwiązaniu.
Zadanie 2.
Udowodnić, że
c
a
b
+√ 2
­1
+√ 2
a2 + 8bc
b + 8ca
c + 8ab
dla dowolnych liczb rzeczywistych dodatnich a, b, c.
√
54
Rozwiązanie
Udowodnimy najpierw, że
4
a
a3
√
(1)
­ 4
4
4 .
2
a + 8bc a 3 + b 3 + c 3
Powyższa nierówność jest równoważna nierówności
4
4
4
2
(a 3 + b 3 + c 3 )2 ­ a 3 (a2 + 8bc).
Korzystając z nierówności między średnią arytmetyczną a średnią geometryczną możemy napisać
4
4
4
4
4
4
4
4
4
4
(a 3 + b 3 + c 3 )2 − (a 3 )2 = (b 3 + c 3 )(a 3 + b 3 + c 3 + a 3 ) ­
2
2
2
1
1
2
­ 2b 3 c 3 · 4a 3 b 3 c 3 = 8a 3 bc.
Zatem
4
4
4
4
2
2
(a 3 + b 3 + c 3 )2 ­ (a 3 )2 + 8a 3 bc = a 3 (a2 + 8bc),
a więc nierówność (1) jest prawdziwa.
W ten sam sposób dowodzimy nierówności
4
4
b3
c
b
c3
oraz √
­ 4
­ 4
4
4 .
4
4
b2 + 8ca a 3 + b 3 + c 3
c2 + 8ab a 3 + b 3 + c 3
Dodając stronami nierówności (1) oraz (2) otrzymujemy żądaną nierówność.
(2)
√
Zadanie 3.
W zawodach matematycznych uczestniczyło 21 dziewcząt i 21 chłopców. Okazało się, że:
• Każdy zawodnik i każda zawodniczka rozwiązał(-a) co najwyżej
sześć zadań.
• Dla każdej dziewczyny i każdego chłopaka istnieje co najmniej
jedno zadanie, które zostało rozwiązane przez oboje z nich.
Udowodnić, że istnieje zadanie, które zostało rozwiązane przez co
najmniej trzy dziewczyny i co najmniej trzech chłopców.
Rozwiązanie
Sposób I
Wprowadźmy następujące oznaczenia: niech C będzie zbiorem chłopców,
a D zbiorem dziewcząt biorących udział w zawodach, Z – zbiorem zadań, Z(c)
oraz Z(d) – zbiorami zadań rozwiązanych odpowiednio przez chłopca c ∈ C
oraz dziewczynę d∈D. Ponadto, niech C(z) oraz D(z) oznaczają odpowiednio
zbiory chłopców oraz dziewcząt, którzy rozwiązali zadanie z ∈ Z. Warunki
zadania można teraz zapisać następująco:
(a) |Z(c)| ¬ 6, |Z(d)| ¬ 6 dla wszystkich c ∈ C oraz d ∈ D;
(b) Z(c) ∩ Z(d) 6= ∅ dla wszystkich c ∈ C oraz d ∈ D.
Mamy wykazać, że istnieje takie zadanie z ∈ Z, dla którego zachodzą nierówności |C(z)| ­ 3, |D(z)| ­ 3. Załóżmy, że teza zadania nie jest prawdziwa.
55
Dostaniemy sprzeczność szacując na dwa sposoby liczbę elementów zbioru
T = {(z, c, d) : z ∈ Z(c) ∩ Z(d)} .
Warunek (b) implikuje, iż
XX
|T | =
(1)
|Z(c) ∩ Z(d)| ­ |C| · |D| = 212 .
c∈C d∈D
Mamy ponadto
X
(2)
|C(z)| =
z∈Z
X
|Z(c)| ¬ 6|C|.
c∈C
Równość w (2) dowodzimy w następujący sposób: Niech χ(c, z) = 1, jeśli c
rozwiązał zadanie z, oraz χ(c, z) = 0 w przeciwnym przypadku. Wówczas wystarczy zamienić kolejność sumowania:
X
|C(z)| =
z∈Z
XX
χ(c, z) =
z∈Z c∈C
XX
χ(c, z) =
c∈C z∈Z
X
|Z(c)|.
c∈C
Analogicznie, mamy
X
(3)
|D(z)| ¬ 6|D|.
z∈Z
Dalej, niech
Z+ = {z ∈ Z : |C(z)| ­ 3}
oraz
Z− = {z ∈ Z : |C(z)| ¬ 2}.
Udowodnimy teraz, że
X
|C(z)| ­ |C| .
z∈Z−
Ustalmy c ∈ C. Na mocy zasady szufladkowej Dirichleta, warunki (a) i (b)
implikują, iż c rozwiązał pewne zadanie z, które zostało rozwiązane przez co
najmniej [21/6] + 1 = 4 dziewczęta. Zatem, na mocy poczynionego założenia
nie wprost, z ∈Z− , a więc każdy chłopiec rozwiązał co najmniej jedno zadanie
ze zbioru Z− . Zatem istotnie
X
|C(z)| ­ |C|.
z∈Z−
Stąd, na mocy (2),
X
z∈Z+
|C(z)| =
X
|C(z)| −
z∈Z
X
|C(z)| ¬ 5|C|.
z∈Z−
Analogicznie dowodzimy nierówności
X
z∈Z−
56
|D(z)| ¬ 5|D|.
I mamy
|T | =
X
z∈Z
¬2
X
|C(z)| · |D(z)| =
X
z∈Z+
z∈Z+
|C(z)| + 2
X
X
|C(z)| · |D(z)| +
|C(z)| · |D(z)| =
z∈Z−
|D(z)| ¬ 10 · |C| + 10 · |D| = 20 · 21.
z∈Z−
Otrzymaliśmy sprzeczność z (1), co kończy dowód.
Sposób II
Użyjemy notacji wprowadzonej w pierwszym rozwiązaniu. Załóżmy, że
dla każdego z ∈ Z mamy |C(z)| ¬ 2 lub |D(z)| ¬ 2. Każde zadanie z ∈ Z malujemy na czerwono, jeśli |C(z)| ¬ 2, bądź na czarno, jeśli |C(z)| ­ 3. Rozważmy
tablicę 21 × 21, której każdy rząd odpowiada jednemu z chłopców, a każda
kolumna jednej z dziewcząt. Dla c ∈ C, d ∈ D bierzemy z ∈ Z(c) ∩ Z(d) i malujemy pole tablicy odpowiadające parze (c, d) na kolor zadania z. Na mocy
zasady szufladkowej Dirichleta, jeden z kolorów zostanie użyty co najmniej
[441/2] + 1 = 221 razy, a zatem w jednym z rzędów znajduje się co najmniej
[221/21] + 1 = 11 czarnych pól lub pewna kolumna zawiera co najmniej 11
czerwonych pól.
Przypuśćmy, że w pewnym rzędzie, odpowiadającym c ∈ C, znajduje się
11 czarnych pól. Spójrzmy na zadania, które były wybierane przy kolorowaniu tych pól. Każde z nich zostało rozwiązane przez co najwyżej dwie
dziewczyny, a więc wśród tych zadań jest co najmniej [11/2]+1 = 6 różnych.
Zatem, na mocy pierwszego warunku, chłopiec c rozwiązał tylko te zadania.
To jednak oznacza, że co najwyżej 12 dziewcząt rozwiązało zadanie, które
zostało rozwiązane przez c. Otrzymujemy sprzeczność z warunkiem drugim.
Analogicznie otrzymujemy sprzeczność, jeśli założymy, że w pewnej kolumnie
znajduje się 11 czerwonych pól.
Zadanie 4.
Niech n będzie liczbą całkowitą nieparzystą większą od 1 i niech
k1 , k2 , . . . , kn będą danymi liczbami całkowitymi. Dla każdej spośród
n! permutacji a = (a1 , a2 , . . . , an ) zbioru {1, 2, . . . , n} przyjmijmy
S(a) =
n
X
ki a i .
i=1
Dowieść, że istnieją dwie różne permutacje b oraz c takie, że liczba
S(b) − S(c) jest podzielna przez n!.
Rozwiązanie
Przypuśćmy, że teza zadania nie jest
prawdziwa; obliczymy na dwa spoP
soby resztę z dzielenia przez n! liczby S(a), gdzie sumowanie rozciąga się
na zbiór wszystkich permutacji a.
P
W wyrażeniu S(a), liczba k1 jest mnożona przez ustaloną liczbę l ze
zbioru {1, 2, . . . , n} dokładnie (n−1)! razy – odpowiada to wszystkim takim
57
permutacjom a, że a1 = l. Zatem współczynnik przy k1 jest równy
(n + 1)!
(n − 1)!(1 + 2 + . . . + n) =
.
2
Taki sam współczynnik stoi przy ki , i = 2, 3, . . . , n, a więc
X
(1)
S(a) =
n
(n + 1)! X
ki .
2
i=1
Z drugiej strony, zgodnie z założeniem nie wprost, liczba S(a)−S(b) dla a 6= b
nie dzieli się przez n!, a więc każda liczba S(a) musi dawać inną resztę przy
dzieleniu przez n!. Ponieważ liczba wszystkich permutacji wynosi n!, więc
zbiór wszystkich reszt, odpowiadających różnym wyborom permutacji a, jest
równy {0, 1, 2, . . . , n!−1} i mamy
(2)
X
S(a) ≡
(n! − 1)n!
2
(mod n!).
Łącząc (1) i (2) otrzymujemy
(3)
n
(n + 1)! X
(n! − 1)n!
ki ≡
2
2
i=1
(mod n!).
Dla n nieparzystych lewa strona (3) przystaje do 0 modulo n!, lecz prawa nie
(gdyż liczba n! − 1 jest nieparzysta). Sprzeczność kończy dowód.
Zadanie 5.
W trójkącie ABC dwusieczna kąta BAC przecina bok BC w punkcie P , a dwusieczna kąta ABC przecina bok CA w punkcie Q. Wia) BAC =60◦ oraz AB+BP =AQ+QB. Wyznaczyć możliwe
domo, że <
miary kątów trójkąta ABC.
Rozwiązanie
) B, co
) C = 12 <
Wykażemy, że BQ = QC. Wiedząc to, stwierdzamy, że <
◦
) C + 2<
) B = 120 daje
w połączeniu z równością <
<
) A = 60◦ ,
<
) B = 80◦ ,
<
) C = 40◦ .
Pozostaje więc wykazać równość BQ = QC.
Na prostej AB wybierzmy taki punkt P 0 , żeby punkty A i P 0 leżały
po różnych stronach punktu B oraz BP 0 = BP (rys. 13). Niech ponadto P 00
będzie punktem symetrycznym do punktu P 0 względem prostej AP . Ponieważ
AQ + QP 00 = AB + BP 0 = AB + BP = AQ + QB ,
więc QP 00 = QB. Zatem zadanie sprowadza się do wykazania, że P 00 = C.
Przypuśćmy, że P 00 6= C. Wówczas punkty B, P , P 00 nie są współliniowe.
Ponadto
) ABC = <
) AP 0 P = <
) AP 00 P ,
<
) QBC = 21 <
co w połączeniu z równością QP 00 = QB daje BP = P P 00 . Z drugiej strony
BP = BP 0 oraz P P 0 = P P 00 . Stąd wynika, że trójkąt BP P 0 jest równoboczny,
58
) ABC =120◦ , czyli <
) ACB =180◦ −<
) ABC−<
) BAC =0◦ . Otrzymana
a zatem <
sprzeczność dowodzi, że P 00 = C.
A
Q
C
B
P
P 00
P0
rys. 13
Zadanie 6.
Niech a, b, c, d będą liczbami całkowitymi spełniającymi warunki
a > b > c > d > 0. Zakładamy, że
ac + bd = (b + d + a − c)(b + d − a + c).
Dowieść, że liczba ab + cd nie jest liczbą pierwszą.
Rozwiązanie
Sposób I
Zauważmy, że
ab + cd = (a + d)c + (b − c)a = m · NWD(a + d, b − c).
Przypuśćmy, wbrew tezie zadania, że liczba ab + cd jest pierwsza. Wobec
tego, albo m = 1, albo NWD(a + d, b − c) = 1. Rozważymy oddzielnie te dwa
przypadki.
(a) m = 1. Wówczas
NWD(a + d, b − c) = ab + cd > ab + cd − (a − b + c + d) =
= (a + d)(c − 1) + (b − c)(a + 1) ­ NWD(a + d, b − c) ,
sprzeczność.
(b) NWD(a+d, b−c)=1. Podstawiając ac+bd =(a+d)b−(b−c)a w miejsce lewej strony danej w treści zadania równości
ac + bd = (b + d + a − c)(b + d − a + c)
otrzymujemy
(a + d)(a − c − d) = (b − c)(b + c + d).
Zatem istnieje taka liczba całkowita dodatnia k, że zachodzą równości
a − c − d = k(b − c)
oraz
b + c + d = k(a + d).
59
Dodając powyższe zależności stronami otrzymujemy
a + b = k(a + b − c + d),
a zatem
k(c − d) = (k − 1)(a + b).
Jeśli k = 1, to c = d i dostajemy sprzeczność. Zatem k ­ 2 i mamy
a+b
k
=
> 2.
2­
k−1 c−d
Otrzymana sprzeczność dowodzi, że liczba ab + cd nie jest pierwsza.
Sposób II
Równość ac + bd = (b + d + a − c)(b + d − a + c) jest równoważna równości
a2 − ac + c2 = b2 + bd + d2 .
(1)
Niech ABCD będzie takim czworokątem, że
) BCD = 120◦ .
) BAD = 60◦ oraz <
AB = a, BC = d, CD = b, AD = c, <
Taki czworokąt istnieje na mocy (1) i twierdzenia cosinusów; wspólna war) ABC = α. Wówczas <
) CDA = 180◦ − α. Stosując
tość w (1) to BD2 . Niech <
twierdzenie cosinusów do trójkątów ABC i ACD otrzymujemy
a2 + d2 − 2ad cos α = AC 2 = b2 + c2 + 2bc cos α.
Zatem 2 cos α = (a2 + d2 − b2 − c2 )/(ad + bc) i stąd
a2 + d2 − b2 − c2 (ab + cd)(ac + bd)
=
.
ad + bc
ad + bc
Na czworokącie ABCD da się opisać okrąg, więc z twierdzenia Ptolemeusza
otrzymujemy
(AC · BD)2 = (ab + cd)2 ,
AC 2 = a2 + d2 − ad ·
skąd wynika, że
(2)
(ac + bd)(a2 − ac + c2 ) = (ab + cd)(ad + bc).
Dalej zauważmy, że
(3)
ab + cd > ac + bd > ad + bc.
Pierwsza nierówność jest równoważna nierówności (a − d)(b − c) > 0, a druga
nierówności (a − b)(c − d) > 0.
Przypuśćmy teraz, że liczba ab+cd jest pierwsza. Wówczas, na mocy (3),
liczby ab + cd i ac + bd są względnie pierwsze. Zatem równość (2) pociąga za
sobą, iż ac+bd dzieli ad+bc. To jednak daje sprzeczność w połączeniu z (3).
Zatem ab + cd nie jest liczbą pierwszą.
æ
60
XXIV Austriacko-Polskie Zawody Matematyczne
Zadanie 1.
Wyznaczyć liczbę całkowitych dodatnich wartości a, dla których istnieją takie nieujemne liczby całkowite x0 , x1 , x2 , . . . , x2001 , że
ax0 =
2001
X
axk .
k=1
Rozwiązanie
Liczba a = 1 nie spełnia warunków zadania, musi więc być a > 1. Odejmijmy liczbę 2001 od obu stron danej równości. Otrzymujemy wówczas
ax0 − 1 − 2000 =
2001
X
(axi − 1).
i=1
Prawa strona jest podzielna przez a − 1, zatem lewa również. Stąd wynika,
że a − 1 jest dzielnikiem liczby 2000. Załóżmy więc, że dla pewnej dodatniej
liczby całkowitej k mamy 2000 = (a − 1)k. Kładziemy
(x0 , x1 , x2 , . . . , x2001 ) =
= (k, 0, 0, . . . , 0, 1, 1, . . . , 1, 2, 2, . . . , 2, . . . , k−1, k−1, . . . , k−1).
|
{z
a
} |
{z } | {z }
a−1
a−1
|
{z
a−1
}
Wówczas
2001
X
axi = a + (a − 1)a + (a − 1)a2 + . . . + (a − 1)ak−1 = ak = ax0 .
i=1
Zatem liczba a spełnia warunki zadania wtedy i tylko wtedy, gdy a − 1 jest
dodatnim dzielnikiem liczby 2000 = 24 · 53 . Takich liczb jest 5 · 4 = 20.
Zadanie 2.
Niech n będzie liczbą całkowitą większą niż 2. Rozwiązać w liczbach rzeczywistych nieujemnych x0 , x1 , x2 , . . . , xn następujący układ
równań:
xk + xk+1 = x2k+2 , k = 1, 2, . . . , n,
gdzie xn+1 = x1 i xn+2 = x2 .
Rozwiązanie
Zauważmy, iż dla dowolnego k = 1, 2, . . . , n mamy xk ¬ 2; w przeciwnym
razie dla pewnego r mielibyśmy xr+2 = max(x1 , x2 , . . . , xn ) > 2, skąd
0 = x2r+2 − xr+1 − xr ­ x2r+2 − 2xr+2 = xr+2 (xr+2 − 2) > 0 .
Sprzeczność.
Podstawmy xk = 1 + dk dla k = 1, 2, . . . , n+2. Otrzymujemy stąd
1 + dk + dk+1 = 2dk+2 + d2k+2 ,
k = 1, 2, . . . , n.
Dodajmy wszystkie równania stronami; w efekcie dostajemy
n=
n
X
d2k .
k=1
61
Ale |dk | ¬ 1; zatem |dk | = 1 i dk + dk+1 = 2dk+2 . Stąd dk = 1 (czyli xk = 2)
dla wszystkich k lub dk = −1 (czyli xk = 0) dla wszystkich k. Bezpośrednio
sprawdzamy, że (x1 , x2 , . . . , xn ) = (0, 0, . . . , 0) oraz (x1 , x2 , . . . , xn ) = (2, 2, . . . , 2)
spełniają rozważany układ równań. Są to więc jedyne rozwiązania.
Zadanie 3.
Niech a, b, c będą długościami boków trójkąta. Udowodnić, że
a + b b + c c + a a3 + b3 + c3
+
+
−
¬ 3.
2<
c
a
b
abc
Rozwiązanie
Pomnóżmy strony nierówności przez abc. Otrzymujemy równoważną nierówność
2abc < ab2 + ba2 + bc2 + cb2 + ca2 + ac2 − a3 − b3 − c3 ¬ 3abc.
Powyższa nierówność wynika z następujących związków:
0 < (−a + b + c)(a − b + c)(a + b − c) =
= ab2 + ba2 + bc2 + cb2 + ca2 + ac2 − a3 − b3 − c3 − 2abc,
oraz
0 ¬ (−a + b + c)(b − c)2 + (a − b + c)(c − a)2 + (a + b − c)(a − b)2 =
= 2(−ab2 − ba2 − bc2 − cb2 − ca2 − ac2 + a3 + b3 + c3 + 3abc).
Zadanie 4.
Udowodnić, że jeśli a, b, c, d są długościami kolejnych boków czworokąta (niekoniecznie wypukłego) o polu S, to spełniona jest nierówność
1
S ¬ (ac + bd).
2
Rozstrzygnąć, dla jakich czworokątów ta nierówność staje się równością.
Rozwiązanie
Sposób I
Niech ABCD będzie czworokątem o polu S i niech AB = a, BC = b,
CD = c, DA = d. Bez straty ogólności możemy przyjąć, że przekątna BD leży
wewnątrz czworokąta ABCD. Niech C 0 będzie punktem symetrycznym do
punktu C względem symetralnej odcinka BD. Wówczas czworokąt ABC 0 D
ma pole S. Oznaczając przez [XY Z] pole trójkąta XY Z uzyskujemy
1
1
) ADC 0 ¬ (ac + bd) .
) ABC 0 ± bd sin <
S = [ABC 0 ] ± [ADC 0 ] = ac sin <
2
2
W powyższych wyrażeniach występuje + lub − w zależności od tego, czy
czworokąt ABC 0 D jest wypukły, czy wklęsły (rys. 14 i rys. 15).
62
A
A
D
B
C0
C
D
B
C0
C
rys. 14
rys. 15
Równość ma miejsce wtedy i tylko wtedy, gdy kąty ABC 0 i ADC 0 są
proste. Warunek ten jest równoważny temu, że punkty A, B, C 0 , D leżą
(w tej właśnie kolejności) na okręgu o średnicy AC 0 . Stąd wynika, że punkty
) ACC 0 = 90◦ oraz CC 0 k BD,
A, B, C, D leżą na jednym okręgu. Ponadto <
skąd wnioskujemy, że BD ⊥ AC.
Również odwrotnie: jeśli czworokąt ABCD o prostopadłych przekątnych
AC i BD da się wpisać w okrąg, to ma miejsce równość w dowodzonej nierówności.
A
B
D
O
C0
C
rys. 16
Sposób II
Niech ABCD będzie czworokątem o polu S i niech AB = a, BC = b,
CD = c, DA = d. Oznaczmy przez α kąt między przekątnymi AC i BD.
Wówczas na mocy nierówności Ptolemeusza (zob. LI Olimpiada Matematyczna, Sprawozdanie Komitetu Głównego, Dodatek D, str. 112, Warszawa
2001) uzyskujemy
ac + bd ­ AC · BD ­ AC · BD · sin α = 2S ,
czyli nierówność, którą należało udowodnić.
Równość zachodzi wtedy i tylko wtedy, gdy na czworokącie ABCD
można opisać okrąg oraz przekątne AC i BD są prostopadłe.
63
Zadanie 5.
Odbywamy zamkniętą podróż skoczkiem po szachownicy, odwiedzając każde z 64 pól dokładnie jeden raz i numerując kolejno odwiedzane pola liczbami od 1 do 64. Następnie wybieramy dodatnie liczby
rzeczywiste x1 , x2 , . . . , x64 . Dla każdego białego pola z numerem i
określamy
q
yi = 1 + x2i − 3 x2i−1 xi+1 ,
a dla każdego czarnego pola z numerem j określamy
yj = 1 + x2j −
q
3
xj−1 x2j+1 ,
gdzie x0 = x64 i x65 = x1 . Udowodnić, że
64
X
yi ­ 48.
i=1
Rozwiązanie
Jeśli pole z numerem i jest białe, to korzystając z nierówności między
średnią arytmetyczną a średnią geometryczną uzyskujemy:
q
2xi−1 + xi+1
√
.
yi = 1 + x2i − 3 x2i−1 xi+1 = 1 + x2i − 3 xi−1 xi−1 xi+1 ­ 1 + x2i −
3
Analogicznie, jeśli pole z numerem i jest czarne, to:
q
xi−1 + 2xi+1
√
yi = 1 + x2i − 3 xi−1 x2i+1 = 1 + x2i − 3 xi−1 xi+1 xi+1 ­ 1 + x2i −
.
3
Zatem mamy
64
X
i=1
Zadanie 6.
yi ­
64
X
64 X
i=1
i=1
(1 + x2i − xi ) =
1 2 3
3
xi −
+
­ 64 · = 48.
2
4
4
Dla ustalonej dodatniej liczby całkowitej k rozważamy ciąg (an )n­0
√
zdefiniowany następująco: a0 = 1, an+1 = an + d k an e dla każdego
n ­ 0, gdzie dxe oznacza najmniejszą liczbę całkowitą nie mniejszą
niż x. Dla każdego k ­1 wyznaczyć zbiór Ak składający się ze wszyst√
kich całkowitych wartości ciągu ( k an )n­0 .
Rozwiązanie
Dla k = 1 mamy an = 2an−1 , skąd an = 2n i zbiór A1 jest zbiorem wszystkich nieujemnych całkowitych potęg liczby 2.
Dla k=2 mamy a2n =(n+1)2 oraz a2n+1 =(n+1)2 +n+1 dla n=0, 1, 2, . . . .
Istotnie: a0 = (0 + 1)2 , a1 = 2 = (0 + 1)2 + 0 + 1, a ponadto
p
√
a2n = a2n−1 + d a2n−1 e = n2 + n + d n2 + n e = (n + 1)2 ,
√
a2n+1 = a2n + d a2n e = (n + 1)2 + n + 1.
√
Mamy więc a2n = n + 1, skąd A2 jest zbiorem wszystkich dodatnich liczb
całkowitych.
Udowodnimy, że dla dowolnej liczby nieparzystej k zachodzi Ak = A1 ,
a dla dowolnej liczby parzystej k mamy Ak = A2 .
64
Przypuśćmy, że an = bk oraz niech M = M (n) będzie największą taką
liczbą całkowitą, że M ­ n i aM < (b + 1)k . Mamy więc
an+1 = an + b
oraz
am+1 = am + b + 1
jeśli tylko n < m ¬ M .
Rozważymy najpierw przypadek, gdy k jest liczbą parzystą. Wówczas
an = bk ≡ 1 (mod b + 1). Zatem dla każdej liczby całkowitej m takiej, że
n < m ¬ M , liczba am jest podzielna przez b+1, a więc aM +1 = (b+1)k . Udowodniliśmy więc, że jeśli b ∈ Ak , to b+1 ∈ Ak . Jednak 1 ∈ Ak , skąd wynika, że
Ak jest zbiorem dodatnich liczb całkowitych.
Przypuśćmy teraz, że k jest liczbą nieparzystą. Wówczas an = bk ≡ −1
(mod b+1), a więc dla każdej liczby całkowitej m takiej, że n < m ¬ M mamy
am ≡−2 (mod b+1). Zatem aM +1 =(b+1)k wtedy i tylko wtedy, gdy b+1=2,
czyli b = 1. W przeciwnym przypadku, aM = (b + 1)k − 2,
aM +1 = (b + 1)k + (b − 1) ≡ −4 (mod b + 2) ,
am+1 = am + (b + 2)
jeśli tylko m ­ M + 1 i am < (b + 2)2 . Zatem am = (b + 2)k dla pewnego m
wtedy i tylko wtedy, gdy b + 2 = 4, czyli b = 2. W przeciwnym razie badamy
wyrazy ciągu modulo b+3, b+4, . . . . Widać, że ai = (b + j)k dla pewnych
całkowitych i > n, j > 0 wtedy i tylko wtedy, gdy b + j = 2j, czyli b + j = 2b.
Zatem udowodniliśmy, że z tego, że b ∈ Ak , wynika, iż 2b ∈ Ak oraz
Ak ∩ {b+1, b+2, . . . , 2b−1} = ∅ .
Ponieważ 1 ∈ Ak , więc Ak jest zbiorem wszystkich całkowitych nieujemnych
potęg liczby 2.
Zadanie 7.
Rozważmy dodatnie liczby całkowite N , w których rozwinięciu dziesiętnym nie występuje cyfra 0 i których suma cyfr S(N ) jest dzielnikiem N .
(a) Udowodnić, że wśród rozważanych liczb N istnieje nieskończenie
wiele takich, w których rozwinięciu dziesiętnym każda z cyfr od
1 do 9 występuje tyle samo razy.
(b) Udowodnić, że dla każdej dodatniej liczby całkowitej k wśród
rozważanych liczb N istnieje liczba k-cyfrowa.
Rozwiązanie
(a) Dla każdej liczby całkowitej dodatniej k skonstruujemy liczbę mającą
9 · 5k−1 cyfr o tej własności, że każda z cyfr 1, 2, . . . , 9 pojawia się 5k−1 razy.
Niech n = 5k . Modyfikujemy liczbę n w następujący sposób: jeśli wśród
ostatnich k cyfr rozwinięcia dziesiętnego liczby n (jeśli n ma l < k cyfr, to
dopisujemy k − l zer z lewej strony) nie występuje cyfra 0, to nic nie robimy. W przeciwnym razie, niech m będzie pierwszym miejscem (patrząc od
prawej strony), na którym stoi cyfra 0. Dodajemy do n liczbę 5k · 10m−1 .
Zmodyfikowana w ten sposób liczba n ma tę własność, że na ostatnich m
65
miejscach nie ma zer. Po skończonej liczbie kroków, otrzymamy taką co najwyżej 2k-cyfrową liczbę n, że wśród ostatnich k cyfr nie występuje cyfra 0.
Zauważmy, że otrzymana liczba n jest podzielna przez 5k .
Dopiszmy zera z lewej strony liczby n tak, aby w sumie było 9·5k−1 cyfr,
a następnie zmodyfikujmy pierwsze 9 · 5k−1 − k cyfr tak, aby w otrzymanym
rozwinięciu każda z cyfr 1, 2, . . . , 9 występowała tyle samo razy. Ponieważ nie
zmienialiśmy ostatnich k cyfr, więc zmodyfikowana liczba (nazwijmy ją Nk )
jest podzielna przez 5k . Jest także podzielna przez 9; suma cyfr liczby Nk
wynosi S(Nk ) = 5k−1 (1 + 2 + . . . + 9) = 9 · 5k . Zatem S(Nk ) | Nk .
(b) Dla k = 1, 2, . . . , 9 liczby 9, 18, 117, 1116, . . . , 111111111 spełniają
warunki zadania.
Dla k = 10, 11, . . . , 40 bierzemy k-cyfrową liczbę . . . 25 o sumie cyfr 45.
Taka liczba istnieje, gdyż kładąc na pierwszych k−2 miejscach cyfry 1 otrzymujemy liczbę o sumie cyfr k − 2 + 2 + 5 = k + 5 ¬ 45, a kładąc na pierwszych
k −2 miejscach cyfry 9 otrzymujemy liczbę o sumie cyfr 9(k −2)+2+5 > 45.
Liczba ta jest podzielna przez 5 i 9, a więc spełnia warunki zadania.
Wreszcie, jeśli k = 41, 42, . . . , 220, to, analogicznie argumentując, można
wykazać, iż liczba . . . 25 o k cyfrach, których suma wynosi 225, jest liczbą
o żądanych własnościach.
Przypuśćmy więc, że k > 220. Wówczas istnieje taka liczba całkowita
dodatnia l, że
5
25
k ¬ 5l ¬ k
3
3
(wynika to stąd, iż iloraz liczby 25/3 przez 5/3 wynosi 5). Ale k > 220, więc
z powyższej nierówności wynika, iż 5l > 366, skąd l > 3 i
3 l
(1)
k­
· 5 > 12l.
25
Analogicznie jak w punkcie (a) konstruujemy liczbę n co najwyżej 2l-cyfrową, podzielną przez 5l , która na ostatnich l miejscach rozwinięcia dziesiętnego nie ma zer. Dopisujemy z lewej strony zera tak, aby w sumie było k
cyfr, a następnie modyfikujemy pierwsze k − l cyfr tak, aby wśród nich nie
występowała cyfra 0 i suma wszystkich k cyfr wynosiła 5l ; jeśli to się uda,
to dostaniemy liczbę o postulowanych własnościach: ze względu na to, że nie
zmieniamy ostatnich l cyfr, otrzymana liczba jest podzielna przez 5l , a więc
także przez sumę swoich cyfr. Należy tylko pokazać, że taka modyfikacja jest
możliwa.
Niech S będzie sumą ostatnich l cyfr. Mamy l < S < 9l. Wystarczy udowodnić, że
(k − l) + S ¬ 5l , 9(k − l) + S ­ 5l .
Pierwsza z nierówności odpowiada liczbie, która na pierwszych k−l miejscach
ma same 1, a druga – liczbie, która na pierwszych k −l miejscach ma same 9.
66
Mamy, korzystając z (1),
2
5
(k − l) + S < k − l + 9l = k + 8l < k + k = k ¬ 5l ,
3
3
oraz
9(k − l) + S > 9(k − l) + l = 9k − 8l =
25
2
25
k + k − 8l > k ­ 5l .
3
3
3
Dowód jest zakończony.
Zadanie 8.
Dany jest graniastosłup prosty o wysokości 1, którego podstawa
jest ośmiokątem foremnym o boku 1, a punkty M1 , M2 , . . . , M10 są
wszystkimi środkami jego ścian. Dla dowolnego punktu P leżącego
wewnątrz graniastosłupa niech Pi oznacza punkt przecięcia prostej
Mi P z jego powierzchnią, różny od Mi . Wybieramy punkt P w ten
sposób, że żaden z punktów Pi nie leży na krawędzi, a na każdej
ścianie znajduje się dokładnie jeden z punktów Pi . Udowodnić, że
10
X
Mi P
i=1
Mi Pi
= 5.
Rozwiązanie
Wykażemy najpierw, że dla każdego i=1, 2, . . . , 10, punkt Pi leży na ścianie przeciwległej do Mi . Przypuśćmy, że tak nie jest; dla pewnego i punkt Pi
nie leży na ścianie S naprzeciw Mi . Wobec tego istnieje taka krawędź k
ściany S, że punkt P oraz ściana S leżą po różnych stronach płaszczyzny W
rozpinanej przez punkt Mi i krawędź k.
Weźmy pod uwagę siedmioelementowy zbiór X złożony z punktu Pi
oraz punktów Pj odpowiadających punktom Mj leżącym po tej samej stronie
płaszczyzny W , co ściana S. Każdy element tego zbioru leży po tej samej stronie płaszczyzny W , co punkt P . Ale tylko 6 ścian rozważanej bryły zawiera
punkty o tej własności. Zatem pewna ściana musi zawierać dwa elementy
zbioru X, co przeczy warunkom zadania.
Przypuśćmy, że Mi , Mj leżą na przeciwległych ścianach Si , Sj rozważanej bryły. Przed chwilą wykazaliśmy, że Pi ∈ Sj , Pj ∈ Si , a zatem proste Mi Pj
i Mj Pi są równoległe oraz
Mj P
Mi Pi − P Pi Mj P
P Pi
Mj P
Mi P
+
=
+
= 1−
+
= 1.
M i Pi M j Pj
M i Pi
M j Pj
M i Pi M j Pj
Ponieważ istnieje 5 par przeciwległych ścian, zatem
10
X
Mi P
i=1
M i Pi
= 5,
czego należało dowieść.
67
Zadanie 9.
Niech n będzie liczbą całkowitą większą od 10 i niech A będzie
zbiorem 2n-elementowym. Załóżmy, że A1 , A2 , . . . , Am są n-elementowymi podzbiorami zbioru A o tej własności, że każde przecięcie
Ai ∩ Aj ∩ Ak dla i 6= j 6= k 6= i ma co najwyżej 1 element. Znaleźć
największą możliwą wartość m w zależności od n.
Rozwiązanie
Jak łatwo sprawdzić, dla m = 4 wystarczy wziąć
A1 = {1, 2, . . . , n} ,
A3 = {1, 3, 5, . . . , 2n−1} ,
A2 = {n+1, n+2, . . . , 2n},
A4 = {2, 4, 6, . . . , 2n} .
Przecięcie dowolnej trójki różnych zbiorów jest zbiorem pustym, a więc warunki zadania są spełnione.
Udowodnimy, że 4 jest szukaną największą wartością m. Przypuśćmy, że
dla m=5 istnieją podzbiory o żądanych własnościach. Dla i∈S ={1, 2, . . . , 2n}
niech xi oznacza liczbę zbiorów Aj , które zawierają i. Wówczas mamy
(1)
5n = |A1 | + |A2 | + . . . + |A5 | =
X
xi ,
i∈S
gdyż w sumie |A1 | + |A2 | + . . . + |A5 | element i ∈ S jest liczony dokładnie xi
razy. Ponadto ma miejsce nierówność
X xi 5
¬
.
(2)
3
3
i∈S
Istotnie, prawa strona to liczba wszystkich trójekzbiorów Aj1 , Aj2 , Aj3 , gdzie
j1 6= j2 6= j3 6= j1 oraz 1 ¬ j1 , j2 , j3 ¬ 5. Liczba x3i to liczba wszystkich takich
trójek, że i ∈ Aj1 ∩ Aj2 ∩ Aj3 . Wystarczy już tylko zauważyć, że każda trójka
(Aj1 , Aj2 , Aj3 ) jest liczona po lewej stronie co najwyżej raz (w przeciwnym
razie zbiór Aj1 ∩ Aj2 ∩ Aj3 byłby co najmniej dwuelementowy, co przeczy
warunkom zadania).
Wystarczy zatem udowodnić, że nie istnieją liczby całkowite nieujemne
x1 , x2 , . . . , xn spełniające warunki (1) i (2).
Dla dowolnej liczby całkowitej nieujemnej k zachodzi
k
k−2 ¬
.
3
Sprawdźmy: powyższa nierówność jest równoważna nierówności
6(k − 2) ¬ k(k − 1)(k − 2),
czyli
0 ¬ (k − 2)(k 2 − k − 6) = (k + 2)(k − 2)(k − 3),
a ta nierówność zachodzi w sposób oczywisty. Zatem, w połączeniu z (1)
oraz (2), otrzymujemy
X X
X
5
xi
10 =
­
­
(xi − 2) =
xi − 4n = n.
3
3
i∈S
i∈S
i∈S
68
Otrzymaliśmy sprzeczność. Dowód jest zakończony.
Zadanie 10. Dany jest ciąg a1 , a2 , . . . , a2010 . Suma każdych 20 kolejnych wyrazów
tego ciągu jest nieujemna. Dla każdego i ∈ {1, 2, . . . , 2009} zachodzi
P
|ai − ai+1 | ¬ 1. Znaleźć najmniejszą możliwą wartość liczby 2010
i=1 ai .
Rozwiązanie
Niech
(a0 , a1 , a2 , . . . , a19 ) =
= (0, −1, −2, −3, −4, −5, −4, −3, −2, −1, 0, 1, 2, 3, 4, 5, 4, 3, 2, 1) .
Ponadto, jeśli liczba n daje z dzielenia przez 20 resztę r, to przyjmujemy
an = ar dla n = 20, 21, . . . , 2010. Wówczas ciąg (an ) spełnia warunki zadania
P
(suma każdych kolejnych 20 wyrazów wynosi 0) oraz 2010
i=1 ai = −25. Zatem
szukana najmniejsza wartość nie przekracza −25. P
Przypuśćmy, że dla pewnego ciągu (an ) mamy 2010
i=1 ai < −25. Na mocy
warunków zadania uzyskujemy
1000
X
ai ­ 0
oraz
2010
X
ai ­ 0,
i=1011
i=1
a więc
(1)
−25 >
2010
X
i=1
ai =
1000
X
i=1
ai +
1010
X
i=1001
ai +
2010
X
ai ­
i=1011
1010
X
ai .
i=1001
Korzystając z tego, że |ai − ai+1 | ¬ 1 dla i = 1, 2, . . . , 2009, otrzymujemy
a1002 ­ a1001 − 1,
a1003 ­ a1002 − 1 ­ a1001 − 2,
a1004 ­ a1003 − 1 ­ a1001 − 3,
a1005 ­ a1004 − 1 ­ a1001 − 4,
a1009 ­ a1010 − 1,
a1008 ­ a1009 − 1 ­ a1010 − 2,
a1007 ­ a1008 − 1 ­ a1010 − 3,
a1006 ­ a1007 − 1 ­ a1010 − 4,
zatem, w połączeniu z (1), −25 > 5(a1001 + a1010 ) − 20, czyli
(2)
a1001 + a1010 < −1.
Z drugiej strony,
a1000 ¬ a1001 + 1,
a999 ¬ a1000 + 1 ¬ a1001 + 2,
a998 ¬ a999 + 1 ¬ a1001 + 3,
a997 ¬ a998 + 1 ¬ a1001 + 4,
69
a996 ¬ a997 + 1 ¬ a1001 + 5,
a1011 ¬ a1010 + 1,
a1012 ¬ a1011 + 1 ¬ a1010 + 2,
a1013 ¬ a1012 + 1 ¬ a1010 + 3,
a1014 ¬ a1013 + 1 ¬ a1010 + 4,
a1015 ¬ a1014 + 1 ¬ a1010 + 5,
zatem, korzystając z (1), (2),
0¬
1015
X
i=996
ai =
1000
X
i=996
ai +
1010
X
i=1001
ai +
1015
X
ai < 5a1001 + 15 − 25 + 5a1010 + 15 < 0.
i=1011
Sprzeczność. Zatem najmniejszą możliwą wartością sumy
æ
70
P2010
i=1
ai jest −25.
XI Zawody Matematyczne Państw Bałtyckich
Zadanie 1.
Na egzamin przygotowano 8 pytań. Każdy uczeń otrzymał 3 z nich.
Żadnych dwóch uczniów nie otrzymało więcej niż jedno wspólne pytanie. Jaka największa możliwa liczba uczniów wzięła udział w egzaminie?
Rozwiązanie
Oznaczmy pytania przez A, B, C, D, E, F , G, H. Wówczas następujące
8 zestawów spełnia warunki zadania: ABC, ADE, AF G, BDG, BF H, CDH,
CEF , EGH. Zatem szukana liczba jest nie mniejsza od 8.
Przypuśćmy, że 4 uczniów otrzymało wspólne pytanie. Wówczas, zgodnie
z warunkami zadania, pozostałe 8 pytań (po 2 od każdego ucznia) musiałoby
być różnych, a stąd byłoby co najmniej 9 pytań – sprzeczność. Zatem każde
pytanie zostało postawione co najwyżej 3 razy, a więc podczas egzaminu
padły co najwyżej 24 pytania. Ponieważ każdy uczeń otrzymywał 3 z nich,
więc było co najwyżej 8 uczniów.
Zadanie 2.
Niech n ­ 2 będzie liczbą całkowitą dodatnią. Rozstrzygnąć, czy
istnieje n takich niepustych parami rozłącznych podzbiorów zbioru
{1, 2, 3, . . .}, że każdą dodatnią liczbę całkowitą można w jednoznaczny sposób przedstawić w postaci sumy co najwyżej n liczb, każdej
z innego podzbioru.
Rozwiązanie
Takie podzbiory istnieją. Oto przykład. Niech
Ak = {2k } dla k = 0, 1, 2, . . . , n−2 oraz An−1 = {2n−1 , 2n−1 · 2, 2n−1 · 3, . . . }.
Powyższe podzbiory spełniają warunki zadania: każdą liczbę całkowitą dodatnią n można w jednoznaczny sposób zapisać w postaci
n=
∞
X
ci (n)2i ,
i=0
gdzie ci (n) ∈ {0, 1}, czyli inaczej
n=
X
ci (n)2i +
i¬n−2
X
ci (n)2i
i>n−2
i jeśli dla i ¬ n − 2 mamy ci (n) = 1, to ci (n)2i ∈ Ai oraz jeśli liczba
X
i>n−2
ci (n)2i = 2n−1
X
ci (n)2i−n+1
i>n−2
nie jest zerem, to należy do zbioru An−1 . Oczywiście to przedstawienie jest
jednoznaczne; przypuśćmy, że n = a0 +a1 +. . .+an−1 oraz ai ∈ Ai ∪{0}. Ponieważ liczba an−1 jest podzielna przez 2n−1 , to liczba a0 +a1 +. . .+an−2 , jako
nie przekraczająca 1+2+22 +. . .+2n−2 = 2n−1 −1, musi być resztą z dzielenia
liczby n przez 2n−2 . Zatem liczby a0 , a1 , . . . , an−2 są wyznaczone jednoznacznie, a więc liczba an−1 = n − (a0 + a1 + . . . + an−2 ) również.
71
Zadanie 3.
Liczby 1, 2, . . . , 49 rozmieszczono w tablicy 7 × 7, po czym obliczono
sumę liczb w każdym wierszu i każdej kolumnie. Niektóre z tych
14 sum są nieparzyste, a pozostałe są parzyste. Niech A oznacza
sumę wszystkich nieparzystych sum, a B sumę wszystkich parzystych
sum. Czy jest możliwe takie rozmieszczenie liczb, że A = B?
Rozwiązanie
Przypuśćmy, że takie rozmieszczenie jest możliwe. Wówczas mamy
2 · (1 + 2 + . . . + 49) = A + B = 2B.
Liczba B, jako suma liczb parzystych, jest parzysta. Wobec tego prawa strona
powyższej równości jest podzielna przez 4. Ale lewa strona nie jest podzielna
przez 4, gdyż 2 · (1 + 2 + . . . + 49) = 2 · 25 · 49. Sprzeczność.
Zadanie 4.
Niech p i q będą dwiema różnymi liczbami pierwszymi. Dowieść, że
p
2p
3p
(q − 1)p
1
+
+
+...+
= (p − 1)(q − 1)
q
q
q
q
2
([x] oznacza największą liczbę całkowitą nie większą niż x).
Rozwiązanie
Sposób I
Liczby p i q są względnie pierwsze, a więc dla k = 1, 2, . . . , q − 1 mamy
kp
(q − k)p
kp
kp
kp
kp
+
=
+ p−
=
+p−1−
= p − 1,
q
q
q
q
q
q
a więc
p
2p
p(q − 1)
2
+
+...+
=
q
q
q
p
(q − 1)p
2p
(q − 2)p
(q − 1)p
p
=
+
+
+
+...+
+
=
q
q
q
q
q
q
= (p − 1)(q − 1),
skąd wynika teza zadania.
Sposób II
Prosta y = (px)/q nie zawiera punktów o współrzędnych całkowitych leżących wewnątrz prostokąta P o wierzchołkach (0, 0), (q, 0), (q, p), (0, p). Dla
x = 1, 2, . . . , q−1 liczba [(px)/q] równa jest liczbie punktów (x, y), gdzie y jest
liczbą całkowitą spełniającą warunek 0 < y < (px)/q. Zatem lewa strona dowodzonej równości jest równa liczbie wszystkich punktów o współrzędnych
całkowitych zawartych między osią OX, a prostą y = (px)/q wewnątrz prostokąta P , co jest równe połowie liczby wszystkich punktów o współrzędnych
całkowitych zawartych w prostokącie P , czyli 21 (p − 1)(q − 1).
72
Zadanie 5.
Każdy spośród danych 2001 punktów na okręgu pokolorowano na
czerwono albo na zielono. W jednym kroku wszystkie punkty są jednocześnie przekolorowywane w następujący sposób: jeśli oba punkty
bezpośrednio sąsiadujące z punktem P mają ten sam kolor co punkt
P , to punkt P nie zmienia koloru, w przeciwnym razie punkt P
zmienia kolor. Rozpoczynając od pokolorowania F1 otrzymujemy kolejno pokolorowania F2 , F3 , . . . . Udowodnić, że istnieje taka liczba
n0 ¬ 1000, że Fn0 = Fn0 +2 . Czy to stwierdzenie jest prawdziwe, jeśli
liczbę 1000 zastąpimy przez 999?
Rozwiązanie
Ponumerujmy dane punkty liczbami 1, 2, . . . , 2001 w ten sposób, że punkty i, j sąsiadują ze sobą, gdy |i − j| = 1 lub {i, j} = {1, 2001}. Powiemy, że
k punktów na okręgu tworzy jednokolorowy segment, jeśli leżą one kolejno
na okręgu i mają ten sam kolor. Dla dowolnego pokolorowania F przez d(F )
będziemy oznaczać maksymalną długość jednokolorowego segmentu.
Dla każdego n zachodzi nierówność d(Fn ) > 1, gdyż liczba 2001 jest nieparzysta. Jeśli d(F1 )=2001, to wszystkie punkty mają ten sam kolor, a zatem
F1 = F2 = F3 = . . . i możemy przyjąć n0 = 1. Niech więc 1 < d(F1 ) < 2001. Niżej
udowodnimy następujące zdania:
(1) Jeśli 3 < d(Fn ) < 2001, to d(Fn+1 ) = d(Fn ) − 2.
(2) Jeśli d(Fn ) = 3, to d(Fn+1 ) = 2.
(3) Jeśli d(Fn ) = 2, to d(Fn+1 ) = d(Fn ) i Fn+2 = Fn .
Wynika stąd, że d(F1000 ) ¬ 2, skąd F1000 = F1002 . Ponadto, jeśli F1 jest
takim pokolorowaniem, że jeden punkt jest czerwony, a pozostałe są zielone,
to d(F1 ) = 2000, a zatem d(F1 ) > d(F2 ) > . . . > d(F1000 ) = 2, co pokazuje, że
dla wszystkich liczb n < 1000 mamy Fn 6= Fn+2 . Zatem nie można zastąpić
liczby 1000 przez 999.
Dowód zdań (1) i (2): Niech {i + 1, i + 2, . . . , i + k} (mod 2001) będzie
najdłuższym jednokolorowym segmentem dla Fn . Wówczas zbiór
{i + 2, i + 3, . . . , i + k − 1}
jest jednokolorowym segmentem dla Fn+1 , a zatem
(4)
d(Fn+1 ) ­ d(Fn ) − 2.
Ponadto, jeśli {i+1, i+2, . . . , i+k} jest najdłuższym jednokolorowym segmentem dla Fn+1 , gdzie k >2, to łatwo zauważyć, że {i, i+1, i+2, . . . , i+k, i+k+1}
jest jednokolorowym segmentem dla Fn . Stąd wynika, że
(5)
d(Fn+1 ) + 2 ¬ d(Fn ),
gdy d(Fn+1 ) > 2.
Stąd wynika prawdziwość zdań (1) i (2).
Dowód zdania (3): Jeśli d(Fn ) = 2, to w następnym kroku wszystkie
punkty zmieniają kolor. Stąd natychmiast otrzymujemy d(Fn )=d(Fn+1 ) oraz
Fn = Fn+2 .
73
Zadanie 6.
Punkty A, B, C, D, E leżą w tej właśnie kolejności na okręgu c
i spełniają AB k EC oraz AC k ED. Prosta styczna do okręgu c w
punkcie E przecina prostą AB w punkcie P . Proste BD i EC przecinają się w punkcie Q. Udowodnić, że AC = P Q.
Rozwiązanie
D
E
Q
P
C
A
B
rys. 17
Wystarczy wykazać, że AP = QC (rys. 17). Wówczas z równoległości
prostych AB, CE wyniknie teza. Ponieważ prosta EP jest styczna do okręgu,
więc w połączeniu z pozostałymi warunkami zadania,
<
) P EA = <
) ECA = <
) CAB = <
) CED = <
) CBQ.
Ponadto,
) P AE − <
) P EA = <
) BAE − <
) P EA =
<
) EP A = 180◦ − <
) EAC = <
) ACD = <
) ABD = <
) BQC.
=<
Również EA = BC, gdyż AB k EC. Zatem trójkąty AP E i CQB są przystające, a więc w szczególności AP = CQ, co było do wykazania.
Zadanie 7.
Dany jest równoległobok ABCD. Okrąg przechodzący przez punkt A
przecina odcinki AB, AC i AD odpowiednio w punktach M , K i N ,
różnych od A. Dowieść, że AB · AM + AD · AN = AK · AC .
Rozwiązanie
) KM N = <
) CAD oraz <
) KN M = <
) KAM = <
) ACD wynika,
Z równości <
że trójkąty KN M i CAD są podobne (rys. 18). Stąd
CD KN
=
,
AD KM
co daje
AB
KN
AB · AM + AD · AN = AD ·
· AM + AN = AD ·
· AM + AN =
AD
KM
AD
=
· (KN · AM + KM · AN ).
KM
74
Na mocy twierdzenia Ptolemeusza, wyrażenie w nawiasie jest równe iloczynowi AK · N M . Jeszcze raz korzystamy z podobieństwa trójkątów KN M
i CAD, otrzymując
AD KM
=
,
AC N M
a stąd
AD
AB · AM + AD · AN =
· AK · N M = AK · AC ,
KM
co było do udowodnienia.
D
C
N
K
A
M
B
rys. 18
Zadanie 8.
Niech ABCD będzie czworokątem wypukłym i niech N będzie środ) AN D = 135◦ . Dowieść, że
kiem boku BC. Ponadto załóżmy, że <
BC
AB + CD + √ ­ AD .
2
Rozwiązanie
Niech B 0 będzie punktem symetrycznym do punktu B względem prostej AN , a C 0 punktem symetrycznym do punktu C względem prostej DN
(rys. 19). Mamy wówczas
<
) B0N C 0 = <
) AN D − <
) AN B 0 − <
) DN C 0 = 135◦ − (<
) AN B + <
) DN C) = 90◦ ,
co w połączeniu z równością B 0 N√
= C 0 N (punkt
N jest środkiem boku BC,
√
0 0
0
więc BN =CN ) daje B C = B N 2 = BC/ 2. Stąd
BC
AD ¬ AB 0 + B 0 C 0 + C 0 D = AB + √ + CD.
2
C
N
B
D
A
C
B0
0
rys. 19
75
Zadanie 9.
Dany jest romb ABCD. Znaleźć zbiór takich punktów P , które leżą
) AP D + <
) BP C = 180◦ .
wewnątrz rombu i spełniają warunek <
Rozwiązanie
Wykażemy, że zbiór punktów P spełniających warunki zadania jest sumą
przekątnych rombu.
Niech Q będzie takim punktem, że czworokąt P QCD jest równoległobokiem (rys. 20). Wówczas czworokąt ABQP jest również równoległobokiem.
) AP D + <
) BP C = 180◦ wynika, że <
) BQC + <
) BP C = 180◦ , więc
Z równości <
) P BC = <
) P QC = <
) P DC,
punkty B, Q, C, P leżą na jednym okręgu. Zatem <
a ponieważ BC = CD, więc dostajemy
<
) CP B = <
) CP D
) CP B + <
) CP D = 180◦ .
lub <
Zatem punkt P leży na odcinku AC lub na odcinku BD.
D
C
P
Q
A
B
rys. 20
Również odwrotnie: dowolny punkt leżący na przekątnej AC spełnia rów) BP C = <
) DP C. Zatem <
) AP D + <
) BP C = 180◦ . Analogicznie dowoność <
dzimy, że równość ta jest spełniona, jeśli punkt P leży na przekątnej BD.
Zadanie 10. W trójkącie ABC dwusieczna kąta BAC przecina bok BC w punk) ADB = 45◦ , wyznaczyć
cie D. Wiedząc, że BD · CD = AD2 oraz <
miary kątów trójkąta ABC.
Rozwiązanie
Sposób I
Niech prosta AD przecina okrąg opisany na trójkącie ABC w punktach
A i E (rys. 21). Niech M będzie środkiem odcinka BC oraz niech O oznacza
środek okręgu opisanego na trójkącie ABC. Wówczas punkty O, M , E są
) ADB = 45◦ mamy <
) M DE = 45◦ ,
współliniowe oraz OE ⊥ BC. Z równości <
◦
) AEO = 45 . Ponieważ AO = EO, więc ostatnia równość implikuje
czyli <
<
) AOE = 90◦ . Stąd uzyskujemy AO k DM .
Z równości BD · CD = AD2 wnioskujemy, że AD = DE, skąd wynika,
że OM = M E. Zatem BO = BE, jak również BO = EO. Trójkąt BOE jest
) BAE = 30◦ , czyli <
) A = 60◦ . Sumując kąty trójkąta
więc równoboczny, skąd <
◦
) B = 105 , skąd w efekcie <
) C = 15◦ .
ABD uzyskujemy <
76
O
A
D
M
B
C
E
rys. 21
Sposób II
) CAD = <
) BAD = α. Mamy wówczas <
) ACD = 45◦ −α, a ponadto
Niech <
◦
<
) ABC = 135 − α. Z warunków zadania wiadomo, iż
AD CD
=
.
BD AD
Korzystając z twierdzenia sinusów w trójkątach ABD, ADC otrzymujemy
AD sin(135◦ − α)
=
BD
sin α
oraz
CD
sin α
=
.
AD sin(45◦ − α)
Zatem
sin2 α = sin(135◦ − α) sin(45◦ − α) = cos(45◦ − α) sin(45◦ − α) =
= 21 sin(90◦ − 2α) = 12 cos 2α = 12 − sin2 α ,
a stąd sin2 α = 41 , czyli α = 30◦ . Zatem kąty trójkąta ABC wynoszą:
<
) A = 60◦ ,
<
) B = 105◦ ,
<
) C = 15◦ .
Zadanie 11. Funkcja f o wartościach rzeczywistych jest określona na zbiorze
wszystkich liczb całkowitych dodatnich. Dla dowolnych liczb całkowitych a > 1, b > 1 oraz d = NWD(a, b) zachodzi równość
b
a
+f
.
f (ab) = f (d) f
d
d
Wyznaczyć wszystkie możliwe wartości f (2001).
Rozwiązanie
Bezpośrednio sprawdzamy, że funkcja tożsamościowo równa 0 oraz funkcja tożsamościowo równa 12 spełniają warunki zadania; stąd mamy dwa rozwiązania: f (2001) = 0, f (2001) = 12 . Wykażemy, że f (2001) nie może przyjmować innych wartości.
Przypuśćmy, że f (2001) 6= 0. Mamy 2001 = 3·667 oraz NWD(3, 667) = 1,
a zatem
f (2001) = f (1)(f (3) + f (667)),
77
skąd wnioskujemy, że f (1)6= 0. Dalej, ponieważ NWD(2001, 2001)=2001, więc
f (20012 ) = f (2001)(2 · f (1)) 6= 0.
(1)
Mamy NWD(2001, 20013 ) = 2001, skąd otrzymujemy
(2)
f (20014 ) = f (2001)(f (1) + f (20012 )) = f (1)f (2001)(1 + 2f (2001)).
Ponadto NWD(20012 , 20012 ) = 20012 . Stąd
f (20014 ) = 2f (1)f (20012 ) = 4f (1)2 f (2001).
Z powyższych dwóch równości wnioskujemy, iż 4f (1) = 1 + 2f (2001), czyli
1
f (2001) = 2f (1) − ,
2
skąd f (1) 6= 14 , gdyż f (2001) 6= 0. Dalej, mamy
(3)
NWD(20012 , 20016 ) = 20012 , NWD(20014 , 20014 ) = 20014 ,
więc, korzystając z (2) uzyskujemy
f (20018 ) = f (20012 )(f (1) + f (20014 )) = f (1)f (20012 )(1 + 2f (20012 )) ,
f (20018 ) = 2f (1)f (20014 ) = 4f (1)2 f (20012 ) ,
skąd 4f (1) = 1 + 2f (20012 ), czyli
1
f (20012 ) = 2f (1) − .
2
Ze związków (1), (3) i (4) wynika, że
(4)
1
1
= 2f (1) 2f (1) −
,
2
2
2f (1) −
czyli
2f (1) −
1
2
2f (1) − 1 = 0,
skąd f (1)=1/2 lub f (1)=1/4. Ale ostatnią równość już wykluczyliśmy. Zatem
f (1) = 1/2 i na mocy (2) mamy f (2001) = 1/2.
Zadanie 12. Niech a1 , a2 , . . . , an będą takimi dodatnimi liczbami rzeczywistymi,
że zachodzą równości
n
X
a3i = 3
i=1
Dowieść, że
n
X
oraz
n
X
a5i = 5.
i=1
ai > 3/2.
i=1
Rozwiązanie
Sposób I
Dla dowolnej liczby rzeczywistej α i dowolnej liczby rzeczywistej dodatniej x zachodzi związek
x5 − 2αx3 + α2 x = x(x2 − α)2 ­ 0.
78
Zatem, podstawiając x = ai , i = 1, 2, . . . , n i sumując otrzymane nierówności
dostajemy, iż dla dowolnej liczby rzeczywistej α zachodzi nierówność
n
X
a5i − 2α
i=1
n
X
a3i + α2
i=1
n
X
ai ­ 0,
i=1
czyli, na mocy warunków zadania,
n
X
ai ­
i=1
6α − 5
.
α2
Pozostaje wykazać, że istnieje taka liczba α, która spełnia nierówność
6α − 5 3
> ,
α2
2
2
czyli równoważnie nierówność 3α − 12α + 10 < 0. To zaś wynika z tego, iż
wyróżnik tego trójmianu jest dodatni: 122 − 4 · 3 · 10 = 24 > 0.
Sposób II
Dla dowolnych liczb rzeczywistych x1 , x2 , . . . , xn , y1 , y2 , . . . , yn zachodzi
nierówność Schwarza
n
X
xi yi ¬
X
n
x2i
1/2 X
n
·
i=1
i=1
yi2
1/2
.
i=1
(Dowód tej nierówności można znaleźć w broszurze L Olimpiada Matematyczna, Sprawozdanie Komitetu Głównego, Dodatek C, str. 112, Warszawa
5/2
1/2
2000). Korzystamy z nierówności Schwarza dla liczb xi = ai oraz yi = ai
uzyskując
2
9=3 =
X
n
a3i
2
i=1
Stąd
=
X
n
5/2
1/2
ai · ai
2
¬
i=1
X
n
a5i
X
n
·
i=1
ai .
i=1
n
X
9 3
9
ai ­ . Wystarczy zauważyć, że > .
5
5 2
i=1
Zadanie 13. Niech a0 , a1 , a2 , . . . będzie ciągiem liczb rzeczywistych spełniającym
warunki
a0 = 1
oraz
an = a[7n/9] + a[n/9]
dla n = 1, 2, 3, . . . .
Dowieść, że istnieje taka liczba całkowita dodatnia k, że
k
ak <
2001!
([x] oznacza największą liczbę całkowitą nie większą niż x).
Rozwiązanie
Wykażemy, że dla pewnego α ∈ ( 21 , 1) zachodzi równość
α
(1)
7
9
α
+
1
9
= 1.
79
W tym celu rozpatrzmy funkcję
x
x
1
− 1, x ∈ R.
9
√
Funkcja f jest ciągła i mamy f (1/2) = ( 7−2)/3 > 0 oraz f (1) = −1/9. Zatem
na mocy własności Darboux funkcja f posiada miejsce zerowe w przedziale
( 21 , 1). To dowodzi istnienia liczby α ∈ ( 12 , 1) spełniającej zależność (1).
Udowodnimy, że zachodzi nierówność
f (x) =
7
9
+
an ¬ 1000 · nα ,
(2)
n = 1, 2, . . . ,
skąd natychmiast wyniknie teza, gdyż istnieje taka liczba naturalna k, że
1000 · k α < k/2001!.
Bezpośrednio sprawdzamy, że nierówność (2) zachodzi dla n = 1, 2, . . . , 8.
Dla n ­ 9 mamy n > [7n/9] ­ 1 oraz n > [n/9] ­ 1, a więc
an = a[7n/9] + a[n/9] ¬ 1000 ·
= 1000 · nα
α
7
9
7n
9
α +
1
9
α
α
+ 1000 ·
n
9
=
= 1000 · nα ,
co kończy dowód.
Zadanie 14. Danych jest 2n kart. Na każdej karcie napisano pewną liczbę rzeczywistą x, przy czym 1 ¬ x ¬ 2 (na różnych kartach mogą być różne
liczby). Udowodnić, że można tak rozdzielić karty na dwa stosy o sumach odpowiednio s1 i s2 , że
n
s1
¬
¬ 1.
n + 1 s2
Rozwiązanie
Oznaczmy liczby napisane na kartach przez x1 , x2 , . . . , x2n , przy czym
x1 ¬ x2 ¬ . . . ¬ x2n . Rozdzielmy karty tak, by
s1 = x1 + x3 + x5 + . . . + x2n−1
s2 = x2 + x4 + x6 + . . . + x2n .
s1
n
Wówczas s1 ¬ s2 . Pozostaje udowodnić lewą nierówność:
¬ . Mamy:
n + 1 s2
oraz
x1 + (x3 + x5 + . . . + x2n−1 )
s1
­
=
s2 x2n + (x2 + x4 + . . . + x2n−2 )
1 + (x3 + x5 + . . . + x2n−1 ) 1 + (x2 + x4 + . . . + x2n−2 )
­
­
=
2 + (x2 + x4 + . . . + x2n−2 ) 2 + (x2 + x4 + . . . + x2n−2 )
1
1
n
= 1−
­ 1−
=
.
2 + (x2 + x4 + . . . + x2n−2 )
2 + (n − 1) n + 1
80
Zadanie 15. Niech a0 , a1 , a2 , . . . będzie ciągiem dodatnich liczb rzeczywistych spełniającym warunek
i · a2i ­ (i + 1) · ai−1 ai+1
dla
i = 1, 2, . . . .
Ponadto niech x i y będą dodatnimi liczbami rzeczywistymi oraz
określmy bi = xai + yai−1 dla i = 1, 2, . . .. Dowieść, że nierówność
i · b2i > (i + 1) · bi−1 bi+1
zachodzi dla wszystkich liczb całkowitych i ­ 2.
Rozwiązanie
Niech i ­ 2 będzie liczbą całkowitą. Wówczas
(i − 1) · a2i−1 ­ i · ai ai−2
(1)
oraz
ia2i ­ (i + 1) · ai+1 ai−1 .
(2)
Mnożąc obie strony nierówności (2) przez x2 otrzymujemy
i · x2 · a2i ­ (i + 1) · x2 · ai+1 ai−1 .
(3)
Na mocy zależności (1) uzyskujemy
a2i−1
i
1
1 i+1
­
= 1+
> 1+ =
,
ai ai−2 i − 1
i−1
i
i
skąd wynika, że
i · y 2 · a2i−1 > (i + 1) · y 2 · ai ai−2 .
(4)
Mnożymy stronami nierówności (1) i (2) i dzielimy obie strony otrzymanej
nierówności przez iai ai−1 . Dostajemy
(i − 1) · ai ai−1 ­ (i + 1) · ai+1 ai−2 .
Dodajemy (i + 1)ai ai−1 do obu stron ostatniej nierówności i mnożymy obie
strony przez xy, otrzymując
(5)
i · 2xy · ai ai−1 ­ (i + 1) · xy · (ai+1 ai−2 + ai ai−1 ).
Dodajemy stronami (3), (4), (5) i otrzymujemy
i · (xai + yai−1 )2 > (i + 1) · (xai+1 + yai )(xai−1 + yai−2 ),
czego należało dowieść.
Zadanie 16. Niech f będzie funkcją o wartościach rzeczywistych określoną na
zbiorze liczb całkowitych dodatnich spełniającą następujący warunek: dla wszystkich n > 1 istnieje taki dzielnik pierwszy p liczby n,
że f (n)=f (n/p)−f (p). Wiedząc, że f (2001)=1, wyznaczyć f (2002).
Rozwiązanie
Udowodnimy indukcyjnie, że zachodzi następujący fakt: jeśli liczba n
jest iloczynem m liczb pierwszych, to f (n) = 21 (2 − m)f (1).
81
Sprawdźmy, czy powyższe zdanie jest prawdziwe dla m = 1. Jeśli n jest
liczbą pierwszą, to jedyną liczbą pierwszą p dzielącą n jest p=n. Stąd wynika,
że f (n) = f (1) − f (n), czyli f (n) = 12 f (1) = 12 (2 − 1)f (1).
Załóżmy więc, że dowodzone zdanie jest prawdziwe dla m−1 i przypuśćmy, że n jest iloczynem m liczb pierwszych. Wówczas, na mocy warunków
zadania, istnieje dzielnik pierwszy p liczby n taki, że f (n) = f (n/p) − f (p).
Liczba n/p jest iloczynem m−1 liczb pierwszych, więc z założenia indukcyjnego f (n/p) = 21 (2−(m−1))f (1) oraz f (p) = 12 f (1). Zatem f (n) = 12 (2−m)f (1).
Liczba 2001 jest iloczynem trzech liczb pierwszych: 2001 = 3 · 23 · 29;
stąd f (2001) = − 21 f (1). Ale z założenia, f (2001) = 1, czyli f (1) = −2. Zatem
f (2002) = f (2 · 7 · 11 · 13) = −1 · f (1) = 2.
Zadanie 17. Niech n będzie liczbą całkowitą dodatnią. Dowieść, że można wybrać
co najmniej 2n−1 + n takich liczb ze zbioru {1, 2, 3, . . . , 2n }, że dla
dowolnych wybranych dwóch różnych liczb x i y, x+y nie dzieli x·y.
Rozwiązanie
Wybieramy liczby 1, 3, 5, . . . , 2n −1 oraz 2, 4, 8, . . . , 2n . Udowodnimy, że
liczby te spełniają warunki zadania. W tym celu rozpatrzymy trzy przypadki.
(i) Jeśli liczby x i y są nieparzyste, to liczba parzysta x + y nie jest
dzielnikiem liczby nieparzystej xy;
(ii) Jeśli liczby x, y są potęgami liczby 2, x = 2a , y = 2b , a, b ­ 1 i na
przykład a < b, to liczba x+y = 2a +2b = 2a (1+2b−a ), posiadająca nieparzysty
dzielnik większy od 1, nie dzieli xy = 2a+b ;
(iii) Jeśli jedna z liczb x, y, na przykład x, jest nieparzysta, a y jest potęgą dwójki, to liczba nieparzysta x + y nie dzieli liczby xy, gdyż największym
nieparzystym dzielnikiem liczby xy jest liczba x.
Wybrane liczby (jest ich 2n−1 + n) spełniają więc warunki zadania.
n
Zadanie 18. Niech am będzie
liczbą nieparzystą. Udowodnić, że liczby a2 + 22
m
n
oraz a2 +22 są względnie pierwsze dla dowolnych dodatnich liczb
całkowitych n i m, przy czym n 6= m.
Rozwiązanie
Zachodzi następująca równość:
k
k
a2 − 22 = (a2
k−1
+ 22
k−1
) · (a2
k−1
− 22
k−1
).
Niech n, m będą pewnymi różnymi dodatnimi liczbami całkowitymi. Bez
straty ogólności możemy przyjąć, że n > m. Mamy
n
n
n
n
n
a2 + 22 = a2 − 22 + 2 · 22 =
= (a2
n−1
+ 22
n
n−1
n
)(a2
n−2
+ 22
m
n−2
m
m
m
m
m
n
) . . . (a2 + 22 )(a2 − 22 ) + 2 · 22 .
Przypuśćmy, że a2 + 22 oraz a2 + 22 mają wspólny dzielnik d > 1. Wówn
czas, na mocy powyższej równości, d dzieli 2·22 , skąd d jest potęgą liczby 2.
82
n
n
A więc liczba 2 dzieli liczbę a2 +22 , która na mocy warunków zadania jest
nieparzysta. Sprzeczność.
Zadanie 19. Jaka jest najmniejsza dodatnia liczba nieparzysta mająca tyle samo
dodatnich dzielników co 360?
Rozwiązanie
Jak łatwo zauważyć, liczba n = pr11 pr22 . . . prkk (gdzie p1 , p2 , . . . , pk są różnymi liczbami pierwszymi) ma dokładnie (r1 +1)(r2 +1) . . . (rk +1) dodatnich
dzielników. Zatem liczba 360 = 23 ·32 ·5 ma 4·3·2 = 24 dzielniki. Niech liczba
n = pr11 pr22 . . . prkk ,
gdzie
p1 < p 2 < . . . < p k
(r1 + 1)(r2 + 1) . . . (rk + 1) = 24,
oraz
będzie szukaną liczbą. Z minimalności liczby n, liczba pi musi być (i+1)-szą
liczbą pierwszą, Ponadto musi zachodzić związek r1 ­ r2 ­ . . . ­ rk ; gdyby dla
pewnych liczb i < j miała miejsce nierówność ri < rj , to zastępując w rozkłar
r
dzie liczby n czynnik pri i pj j przez czynnik pi j prj i otrzymalibyśmy mniejszą
liczbę mającą 24 dzielniki. Mamy następujące rozkłady liczby 24:
3 · 2 · 2 · 2,
4 · 3 · 2,
6 · 4,
6 · 2 · 2,
12 · 2,
24 .
Pozostaje tylko sprawdzić, że pierwszy rozkład daje najmniejszą liczbę równą
32 · 5 · 7 · 11 = 3465.
Zadanie 20. Z ciągu (a, b, c, d) liczb całkowitych możemy uzyskać w jednym kroku
jeden z ciągów
(c, d, a, b),
(b, a, d, c),
(a + nc, b + nd, c, d),
(a + nb, b, c + nd, d)
dla dowolnej liczby całkowitej n, przy czym liczba n może być w każdym kroku inna. Rozstrzygnąć, czy przy pomocy pewnej liczby kroków można otrzymać ciąg (3, 4, 5, 7) z ciągu (1, 2, 3, 4).
Rozwiązanie
Nie można.
Ciągowi (p, q, r, s) przyporządkujmy liczbę F = F (p, q, r, s) = |ps − qr|.
Wówczas liczba F nie zmienia się wskutek operacji opisanych w treści zadania. Wystarczy już tylko zauważyć, że F (1, 2, 3, 4) = 2 i F (3, 4, 5, 7) = 1.
83
Download